You are on page 1of 100

Chapter 2 Solving Linear Equations 193

x ≤ 21 or x ≥ −12
2

Graph each
solution.

Graph numbers
that make either
inequality true.

The solution covers all real numbers.

(−∞, ∞)

TRY IT : : 2.131

Solve the compound inequality. Graph the solution and write the solution in interval notation: 3 x − 7 ≤ −1 or
5
1 (x + 6) ≥ −2.
3

TRY IT : : 2.132

Solve the compound inequality. Graph the solution and write the solution in interval notation: 3 x − 3 ≤ 3 or
4
2 (x + 10) ≥ 0.
5

Solve Applications with Compound Inequalities


Situations in the real world also involve compound inequalities. We will use the same problem solving strategy that we
used to solve linear equation and inequality applications.
Recall the problem solving strategies are to first read the problem and make sure all the words are understood. Then,
identify what we are looking for and assign a variable to represent it. Next, restate the problem in one sentence to make
it easy to translate into a compound inequality. Last, we will solve the compound inequality.

EXAMPLE 2.67

Due to the drought in California, many communities have tiered water rates. There are different rates for Conservation
Usage, Normal Usage and Excessive Usage. The usage is measured in the number of hundred cubic feet (hcf) the property
owner uses.
During the summer, a property owner will pay $24.72 plus $1.54 per hcf for Normal Usage. The bill for Normal Usage
would be between or equal to $57.06 and $171.02. How many hcf can the owner use if he wants his usage to stay in the
normal range?
194 Chapter 2 Solving Linear Equations

Solution

Identify what we are The number of hcf he can use and stay in the “normal usage” billing range.
looking for.

Name what we are looking Let x = the number of hcf he can use.
for.

Translate to an inequality. Bill is $24.72 plus $1.54 times the number of hcf he uses or 24.72 + 1.54x.

Solve the inequality.

Answer the question. The property owner can use 21–95 hcf and still fall within the “normal
usage” billing range.

TRY IT : : 2.133

Due to the drought in California, many communities now have tiered water rates. There are different rates for
Conservation Usage, Normal Usage and Excessive Usage. The usage is measured in the number of hundred cubic
feet (hcf) the property owner uses.
During the summer, a property owner will pay $24.72 plus $1.32 per hcf for Conservation Usage. The bill for
Conservation Usage would be between or equal to $31.32 and $52.12. How many hcf can the owner use if she
wants her usage to stay in the conservation range?

TRY IT : : 2.134

Due to the drought in California, many communities have tiered water rates. There are different rates for
Conservation Usage, Normal Usage and Excessive Usage. The usage is measured in the number of hundred cubic
feet (hcf) the property owner uses.
During the winter, a property owner will pay $24.72 plus $1.54 per hcf for Normal Usage. The bill for Normal Usage
would be between or equal to $49.36 and $86.32. How many hcf will he be allowed to use if he wants his usage to
stay in the normal range?

MEDIA : :
Access this online resource for additional instruction and practice with solving compound inequalities.
• Compound inequalities (https://openstax.org/l/37compinequalit)

This OpenStax book is available for free at http://cnx.org/content/col12119/1.5


Chapter 2 Solving Linear Equations 195

2.6 EXERCISES
Practice Makes Perfect
Solve Compound Inequalities with “and”
In the following exercises, solve each inequality, graph the solution, and write the solution in interval notation.
376. x < 3 and x ≥ 1 377. x ≤ 4 and x > −2 378. x ≥ −4 and x ≤ −1

379. x > −6 and x < −3 380. 5x − 2 < 8 and 6x + 9 ≥ 3 381. 4x − 1 < 7 and 2x + 8 ≥ 4

382.4x + 6 ≤ 2 and 383. 4x − 2 ≤ 4 and 384.2x − 11 < 5 and


2x + 1 ≥ −5 7x − 1 > −8 3x − 8 > −5

385.7x − 8 < 6 and 386. 4(2x − 1) ≤ 12 and 387.5(3x − 2) ≤ 5 and


5x + 7 > −3 2(x + 1) < 4 3(x + 3) < 3

388.3(2x − 3) > 3 and 389.−3(x + 4) < 0 and 390. 1 (3x − 4) ≤ 1 and


−1(3x − 1) ≤ 7 2
4(x + 5) ≥ 4
1 (x + 6) ≤ 4
3

391. 3 (x − 8) ≤ 3 and 392. 5x − 2 ≤ 3x + 4 and 393.3 x − 5 ≥ −2 and


4 3x − 4 ≥ 2x + 1 4
1 (x − 5) ≤ 3 −3(x + 1) ≥ 6
5

394.2 x − 6 ≥ −4 and 1 (x − 6) + 2 < −5 and


395. 396. −5 ≤ 4x − 1 < 7
3 2
−4(x + 2) ≥ 0 2
4− x<6
3

397. −3 < 2x − 5 ≤ 1 398. 5 < 4x + 1 < 9 399. −1 < 3x + 2 < 8

400. −8 < 5x + 2 ≤ −3 401. −6 ≤ 4x − 2 < −2

Solve Compound Inequalities with “or”


In the following exercises, solve each inequality, graph the solution on the number line, and write the solution in interval notation.
402. x ≤ −2 or x > 3 403. x ≤ −4 or x > −3 404. x < 2 or x ≥ 5

405. x < 0 or x ≥ 4 406. 2 + 3x ≤ 4 or 407.4 − 3x ≤ −2 or


5 − 2x ≤ −1 2x − 1 ≤ −5

408.2(3x − 1) < 4 or 409. 3(2x − 3) < −5 or 410. 3 x − 2 > 4 or 4(2 − x) > 0


3x − 5 > 1 4x − 1 > 3 4

411. 2 x − 3 > 5 or 3(5 − x) > 6 412. 3x − 2 > 4 or 5x − 3 ≤ 7 413.2(x + 3) ≥ 0 or


3 3(x + 4) ≤ 6
196 Chapter 2 Solving Linear Equations

414. 1 x − 3 ≤ 4 or 415. 3 x + 2 ≤ −1 or
2 4
1 (x − 6) ≥ −2 1 (x + 8) ≥ −3
3 2

Mixed practice
In the following exercises, solve each inequality, graph the solution on the number line, and write the solution in interval notation.
416.3x + 7 ≤ 1 and 417. 6(2x − 1) > 6 and 418. 4 − 7x ≥ −3 or
2x + 3 ≥ −5 5(x + 2) ≥ 0 5(x − 3) + 8 > 3

419. 1 x − 5 ≤ 3 or 420. −5 ≤ 2x − 1 < 7 1 (x − 5) + 6 < 4 and


421.
2 5
1 (x − 8) ≥ −3 2
3− x<5
4 3

422.4x − 2 > 6 or 423. 6x − 3 ≤ 1 and 424. −2(3x − 4) ≤ 2 and


3x − 1 ≤ −2 5x − 1 > −6 −4(x − 1) < 2

425. −5 ≤ 3x − 2 ≤ 4

Solve Applications with Compound Inequalities


In the following exercises, solve.
426. Penelope is playing a number game with her sister 427. Gregory is thinking of a number and he wants
June. Penelope is thinking of a number and wants June his sister Lauren to guess the number. His first clue is
to guess it. Five more than three times her number is that six less than twice his number is between four and
between 2 and 32. Write a compound inequality that forty-two. Write a compound inequality that shows the
shows the range of numbers that Penelope might be range of numbers that Gregory might be thinking of.
thinking of.

428. Andrew is creating a rectangular dog run in his 429. Elouise is creating a rectangular garden in her
back yard. The length of the dog run is 18 feet. The back yard. The length of the garden is 12 feet. The
perimeter of the dog run must be at least 42 feet and perimeter of the garden must be at least 36 feet and no
no more than 72 feet. Use a compound inequality to more than 48 feet. Use a compound inequality to find
find the range of values for the width of the dog run. the range of values for the width of the garden.

Everyday Math
430. Blood Pressure A person’s blood pressure is 431. Body Mass Index (BMI) is a measure of body fat is
measured with two numbers. The systolic blood determined using your height and weight.
pressure measures the pressure of the blood on the
arteries as the heart beats. The diastolic blood ⓐ Let x be your BMI. Research and then write the
pressure measures the pressure while the heart is compound inequality to show the BMI range for you to
resting. be considered normal weight.

ⓐ Let x be your systolic blood pressure. Research and ⓑ Research a BMI calculator and determine your BMI.
then write the compound inequality that shows you Is it a solution to the inequality in part (a)?
what a normal systolic blood pressure should be for
someone your age.

ⓑ Let y be your diastolic blood pressure. Research and


then write the compound inequality that shows you
what a normal diastolic blood pressure should be for
someone your age.

This OpenStax book is available for free at http://cnx.org/content/col12119/1.5


Chapter 2 Solving Linear Equations 197

Writing Exercises
432. In your own words, explain the difference between 433. Explain the steps for solving the compound
the properties of equality and the properties of inequality 2 − 7x ≥ −5 or 4(x − 3) + 7 > 3.
inequality.

Self Check

ⓐ After completing the exercises, use this checklist to evaluate your mastery of the objectives of this section.

ⓑ What does this checklist tell you about your mastery of this section? What steps will you take to improve?
198 Chapter 2 Solving Linear Equations

2.7
Solve Absolute Value Inequalities
Learning Objectives
By the end of this section, you will be able to:
Solve absolute value equations
Solve absolute value inequalities with “less than”
Solve absolute value inequalities with “greater than”
Solve applications with absolute value

Be Prepared!

Before you get started, take this readiness quiz.


1. Evaluate: −|7|.
If you missed this problem, review Example 1.12.
2. Fill in <, >, or = for each of the following pairs of numbers.
ⓐ |−8|___ − |−8| ⓑ 12___ − |−12| ⓒ |−6|___ − 6 ⓓ −(−15)___ − |−15|
If you missed this problem, review Example 1.12.
3. Simplify: 14 − 2|8 − 3(4 − 1)|.
If you missed this problem, review Example 1.13.

Solve Absolute Value Equations


As we prepare to solve absolute value equations, we review our definition of absolute value.

Absolute Value

The absolute value of a number is its distance from zero on the number line.
The absolute value of a number n is written as |n| and |n| ≥ 0 for all numbers.
Absolute values are always greater than or equal to zero.

We learned that both a number and its opposite are the same distance from zero on the number line. Since they have the
same distance from zero, they have the same absolute value. For example:
−5 is 5 units away from 0, so |−5| = 5.
5 is 5 units away from 0, so |5| = 5.
Figure 2.6 illustrates this idea.

Figure 2.6 The numbers 5 and −5 are both five


units away from zero.

For the equation |x| = 5, we are looking for all numbers that make this a true statement. We are looking for the numbers
whose distance from zero is 5. We just saw that both 5 and −5 are five units from zero on the number line. They are the
solutions to the equation.
If | x| = 5
then x = −5 or x = 5
The solution can be simplified to a single statement by writing x = ±5. This is read, “x is equal to positive or negative 5”.
We can generalize this to the following property for absolute value equations.

This OpenStax book is available for free at http://cnx.org/content/col12119/1.5


Chapter 2 Solving Linear Equations 199

Absolute Value Equations

For any algebraic expression, u, and any positive real number, a,


if |u| = a
then u = −a or u = a
Remember that an absolute value cannot be a negative number.

EXAMPLE 2.68

Solve: ⓐ | x| = 8 ⓑ |y| = −6 ⓒ |z| = 0


Solution

| x| = 8
Write the equivalent equations. x = −8 or x = 8
x = ±8


|y| = −6
No solution
Since an absolute value is always positive, there are no solutions to this equation.


|z| = 0
Write the equivalent equations. z = −0 or z = 0
Since −0 = 0, z=0
Both equations tell us that z = 0 and so there is only one solution.

TRY IT : : 2.135 Solve: ⓐ | x| = 2 ⓑ |y| = −4 ⓒ |z| = 0

TRY IT : : 2.136 Solve: ⓐ |x| = 11 ⓑ |y| = −5 ⓒ |z| = 0

To solve an absolute value equation, we first isolate the absolute value expression using the same procedures we used to
solve linear equations. Once we isolate the absolute value expression we rewrite it as the two equivalent equations.

EXAMPLE 2.69 HOW TO SOLVE ABSOLUTE VALUE EQUATIONS

Solve |5x − 4| − 3 = 8.
Solution
200 Chapter 2 Solving Linear Equations

TRY IT : : 2.137 Solve: |3x − 5| − 1 = 6.

TRY IT : : 2.138 Solve: |4x − 3| − 5 = 2.

The steps for solving an absolute value equation are summarized here.

HOW TO : : SOLVE ABSOLUTE VALUE EQUATIONS.

Step 1. Isolate the absolute value expression.


Step 2. Write the equivalent equations.
Step 3. Solve each equation.
Step 4. Check each solution.

EXAMPLE 2.70

Solve 2|x − 7| + 5 = 9.
Solution

2|x − 7| + 5 = 9

Isolate the absolute value expression. 2|x − 7| = 4

|x − 7| = 2

Write the equivalent equations. x − 7 = −2 or x − 7 = 2

Solve each equation. x=5 or x=9

This OpenStax book is available for free at http://cnx.org/content/col12119/1.5


Chapter 2 Solving Linear Equations 201

Check:

TRY IT : : 2.139 Solve: 3|x − 4| − 4 = 8.

TRY IT : : 2.140 Solve: 2|x − 5| + 3 = 9.

Remember, an absolute value is always positive!

EXAMPLE 2.71

Solve:
|23 x − 4| + 11 = 3.
Solution

|23 x − 4| + 11 = 3
Isolate the absolute value term.
An absolute value cannot be negative.
|No23solution
|
x − 4 = −8

TRY IT : : 2.141
Solve:
|34 x − 5| + 9 = 4.
TRY IT : : 2.142
Solve:
|56 x + 3| + 8 = 6.
Some of our absolute value equations could be of the form |u| = |v| where u and v are algebraic expressions. For
example, |x − 3| = |2x + 1|.
How would we solve them? If two algebraic expressions are equal in absolute value, then they are either equal to each
other or negatives of each other. The property for absolute value equations says that for any algebraic expression, u, and
a positive real number, a, if |u| = a, then u = −a or u = a.

This tell us that


if |u| = |v|
then u = −v or u=v
This leads us to the following property for equations with two absolute values.

Equations with Two Absolute Values

For any algebraic expressions, u and v,


if |u| = |v|
then u = −v or u = v
When we take the opposite of a quantity, we must be careful with the signs and to add parentheses where needed.
202 Chapter 2 Solving Linear Equations

EXAMPLE 2.72

Solve: |5x − 1| = |2x + 3|.


Solution
|5x − 1| = |2x + 3|
Write the equivalent equations. 5x − 1 = −(2x + 3) or 5x − 1 = 2x + 3
Solve each equation. 5x − 1 = −2x − 3 or 3x − 1 = 3
7x − 1 = −3 3x = 4
7x = −2 x = 4
3
x = −2 or x = 4
7 3
Check.
We leave the check to you.

TRY IT : : 2.143 Solve: |7x − 3| = |3x + 7|.

TRY IT : : 2.144 Solve: |6x − 5| = |3x + 4|.

Solve Absolute Value Inequalities with “Less Than”


Let’s look now at what happens when we have an absolute value inequality. Everything we’ve learned about solving
inequalities still holds, but we must consider how the absolute value impacts our work.
Again we will look at our definition of absolute value. The absolute value of a number is its distance from zero on the
number line. For the equation | x| = 5, we saw that both 5 and −5 are five units from zero on the number line. They are
the solutions to the equation.
| x| = 5
x = −5 or x=5
What about the inequality |x| ≤ 5 ? Where are the numbers whose distance is less than or equal to 5? We know −5 and
5 are both five units from zero. All the numbers between −5 and 5 are less than five units from zero. See Figure 2.7.

Figure 2.7

In a more general way, we can see that if |u| ≤ a, then −a ≤ u ≤ a. See Figure 2.8.

Figure 2.8

This result is summarized here.

Absolute Value Inequalities with < or ≤

For any algebraic expression, u, and any positive real number, a,

This OpenStax book is available for free at http://cnx.org/content/col12119/1.5


Chapter 2 Solving Linear Equations 203

if |u| < a, then −a < u < a


if |u| ≤ a, then −a ≤ u ≤ a
After solving an inequality, it is often helpful to check some points to see if the solution makes sense. The graph of
the solution divides the number line into three sections. Choose a value in each section and substitute it in the original
inequality to see if it makes the inequality true or not. While this is not a complete check, it often helps verify the solution.

EXAMPLE 2.73

Solve |x| < 7. Graph the solution and write the solution in interval notation.
Solution

Write the equivalent inequality.

Graph the solution.

Write the solution using interval


notation.

Check:
To verify, check a value in each section of the number line showing the solution. Choose numbers such as −8, 1, and 9.

TRY IT : : 2.145 Graph the solution and write the solution in interval notation: |x| < 9.

TRY IT : : 2.146 Graph the solution and write the solution in interval notation: |x| < 1.

EXAMPLE 2.74

Solve |5x − 6| ≤ 4. Graph the solution and write the solution in interval notation.

Solution

Step 1. Isolate the absolute value expression. |5x − 6| ≤ 4


It is isolated.

Step 2. Write the equivalent compound inequality. −4 ≤ 5x − 6 ≤ 4

Step 3. Solve the compound inequality. 2 ≤ 5x ≤ 10


2≤x≤2
5

Step 4. Graph the solution.

Step 5. Write the solution using interval notation. ⎡2 ⎤


⎣ 5 , 2⎦
204 Chapter 2 Solving Linear Equations

Check:
The check is left to you.

TRY IT : : 2.147 Solve |2x − 1| ≤ 5. Graph the solution and write the solution in interval notation:

TRY IT : : 2.148 Solve |4x − 5| ≤ 3. Graph the solution and write the solution in interval notation:

HOW TO : : SOLVE ABSOLUTE VALUE INEQUALITIES WITH < OR ≤.

Step 1. Isolate the absolute value expression.


Step 2. Write the equivalent compound inequality.
|u| < a is equivalent to −a < u < a
|u| ≤ a is equivalent to −a ≤ u ≤ a
Step 3. Solve the compound inequality.
Step 4. Graph the solution
Step 5. Write the solution using interval notation.

Solve Absolute Value Inequalities with “Greater Than”


What happens for absolute value inequalities that have “greater than”? Again we will look at our definition of absolute
value. The absolute value of a number is its distance from zero on the number line.
We started with the inequality |x| ≤ 5. We saw that the numbers whose distance is less than or equal to five from zero
on the number line were −5 and 5 and all the numbers between −5 and 5. See Figure 2.9.

Figure 2.9

Now we want to look at the inequality |x| ≥ 5. Where are the numbers whose distance from zero is greater than or equal
to five?
Again both −5 and 5 are five units from zero and so are included in the solution. Numbers whose distance from zero is
greater than five units would be less than −5 and greater than 5 on the number line. See Figure 2.10.

Figure 2.10

In a more general way, we can see that if |u| ≥ a, then u ≤ −a or u ≤ a. See Figure 2.11.

This OpenStax book is available for free at http://cnx.org/content/col12119/1.5


Chapter 2 Solving Linear Equations 205

Figure 2.11

This result is summarized here.

Absolute Value Inequalities with > or ≥

For any algebraic expression, u, and any positive real number, a,


if |u| > a, then u < − a or u > a
if |u| ≥ a, then u ≤ −a or u ≥ a

EXAMPLE 2.75

Solve |x| > 4. Graph the solution and write the solution in interval notation.
Solution

| x| > 4

Write the equivalent inequality. x < −4 or x > 4

Graph the solution.

Write the solution using interval (−∞, −4) ∪ (4, ∞)


notation.

Check:

To verify, check a value in each section of the number line showing the solution. Choose numbers such as −6, 0, and 7.

TRY IT : : 2.149 Solve |x| > 2. Graph the solution and write the solution in interval notation.

TRY IT : : 2.150 Solve |x| > 1. Graph the solution and write the solution in interval notation.

EXAMPLE 2.76

Solve |2x − 3| ≥ 5. Graph the solution and write the solution in interval notation.
206 Chapter 2 Solving Linear Equations

Solution

|2x − 3| ≥ 5

Step 1. Isolate the absolute value expression. It is


isolated.

Step 2. Write the equivalent compound inequality. 2x − 3 ≤ −5 or 2x − 3 ≥ 5

Step 3. Solve the compound inequality. 2x ≤ −2 or 2x ≥ 8


x ≤ −1 or x ≥ 4

Step 4. Graph the solution.

Step 5. Write the solution using interval notation. (−∞, −1] ∪ [4, ∞)

Check:
The check is left to you.

TRY IT : : 2.151 Solve |4x − 3| ≥ 5. Graph the solution and write the solution in interval notation.

TRY IT : : 2.152 Solve |3x − 4| ≥ 2. Graph the solution and write the solution in interval notation.

HOW TO : : SOLVE ABSOLUTE VALUE INEQUALITIES WITH > OR ≥.

Step 1. Isolate the absolute value expression.


Step 2. Write the equivalent compound inequality.
|u| > a is equivalent to u < − a or u > a
|u| ≥ a is equivalent to u ≤ −a or u ≥ a
Step 3. Solve the compound inequality.
Step 4. Graph the solution
Step 5. Write the solution using interval notation.

Solve Applications with Absolute Value


Absolute value inequalities are often used in the manufacturing process. An item must be made with near perfect
specifications. Usually there is a certain tolerance of the difference from the specifications that is allowed. If the difference
from the specifications exceeds the tolerance, the item is rejected.
|actual-ideal| ≤ tolerance
EXAMPLE 2.77

The ideal diameter of a rod needed for a machine is 60 mm. The actual diameter can vary from the ideal diameter by
0.075 mm. What range of diameters will be acceptable to the customer without causing the rod to be rejected?

This OpenStax book is available for free at http://cnx.org/content/col12119/1.5


Chapter 2 Solving Linear Equations 207

Solution
Let x = the actual measurement.
Use an absolute value inequality to express this situation. |actual-ideal| ≤ tolerance
|x − 60| ≤ 0.075
Rewrite as a compound inequality. −0.075 ≤ x − 60 ≤ 0.075
Solve the inequality. 59.925 ≤ x ≤ 60.075
Answer the question. The diameter of the rod can be between
59.925 mm and 60.075 mm.

TRY IT : : 2.153

The ideal diameter of a rod needed for a machine is 80 mm. The actual diameter can vary from the ideal diameter
by 0.009 mm. What range of diameters will be acceptable to the customer without causing the rod to be rejected?

TRY IT : : 2.154

The ideal diameter of a rod needed for a machine is 75 mm. The actual diameter can vary from the ideal diameter
by 0.05 mm. What range of diameters will be acceptable to the customer without causing the rod to be rejected?

MEDIA : :
Access this online resource for additional instruction and practice with solving linear absolute value equations and
inequalities.
• Solving Linear Absolute Value Equations and Inequalities (https://openstax.org/l/37solvlinabsol)
208 Chapter 2 Solving Linear Equations

2.7 EXERCISES
Practice Makes Perfect
Solve Absolute Value Equations
In the following exercises, solve.

434. ⓐ | x| = 6 ⓑ |y| = −3 435. ⓐ | x| = 4 ⓑ |y| = −5 436. ⓐ | x| = 7 ⓑ |y| = −11


ⓒ |z| = 0 ⓒ |z| = 0 ⓒ |z| = 0

437. ⓐ | x| = 3 ⓑ |y| = −1 438. |2x − 3| − 4 = 1 439. |4x − 1| − 3 = 0

ⓒ |z| = 0

440. |3x − 4| + 5 = 7 441. |4x + 7| + 2 = 5 442. 4|x − 1| + 2 = 10

443. 3|x − 4| + 2 = 11 444. 3|4x − 5| − 4 = 11 445. 3|x + 2| − 5 = 4

446. −2|x − 3| + 8 = −4 447. −3|x − 4| + 4 = −5 448.


|34 x − 3| + 7 = 2
449.
| |
3x − 2 + 5 = 2
5
450.
|12 x + 5| + 4 = 1 451.
|14 x + 3| + 3 = 1
452. |3x − 2| = |2x − 3| 453. |4x + 3| = |2x + 1| 454. |6x − 5| = |2x + 3|

455. |6 − x| = |3 − 2x|

Solve Absolute Value Inequalities with “less than”


In the following exercises, solve each inequality. Graph the solution and write the solution in interval notation.
456. | x| < 5 457. | x| < 1 458. | x| ≤ 8

459. | x| ≤ 3 460. |3x − 3| ≤ 6 461. |2x − 5| ≤ 3

462. |2x + 3| + 5 < 4 463. |3x − 7| + 3 < 1 464. |4x − 3| < 1

465. |6x − 5| < 7 466. |x − 4| ≤ −1 467. |5x + 1| ≤ −2

Solve Absolute Value Inequalities with “greater than”


In the following exercises, solve each inequality. Graph the solution and write the solution in interval notation.
468. | x| > 3 469. | x| > 6 470. | x| ≥ 2

471. | x| ≥ 5 472. |3x − 8| > −1 473. |x − 5| > −2

474. |3x − 2| > 4 475. |2x − 1| > 5 476. |x + 3| ≥ 5

477. |x − 7| ≥ 1 478. 3|x| + 4 ≥ 1 479. 5|x| + 6 ≥ 1

In the following exercises, solve. For each inequality, also graph the solution and write the solution in interval notation.
480. 2|x + 6| + 4 = 8 481. |3x − 4| ≥ 2 482. |6x − 5| = |2x + 3|

This OpenStax book is available for free at http://cnx.org/content/col12119/1.5


Chapter 2 Solving Linear Equations 209

483. |4x − 3| < 5 484. |2x − 5| + 2 = 3 485. |3x + 1| − 3 = 7

486. |7x + 2| + 8 < 4 487. 5|2x − 1| − 3 = 7 488. |x − 7| > −3

489. |8 − x| = |4 − 3x|

Solve Applications with Absolute Value


In the following exercises, solve.
490. A chicken farm ideally produces 200,000 eggs per 491. An organic juice bottler ideally produces 215,000
day. But this total can vary by as much as 25,000 eggs. bottle per day. But this total can vary by as much as
What is the maximum and minimum expected 7,500 bottles. What is the maximum and minimum
production at the farm? expected production at the bottling company?

492. In order to insure compliance with the law, Miguel 493. At Lilly’s Bakery, the ideal weight of a loaf of bread
routinely overshoots the weight of his tortillas by 0.5 is 24 ounces. By law, the actual weight can vary from
gram. He just received a report that told him that he the ideal by 1.5 ounces. What range of weight will be
could be losing as much as $100,000 per year using acceptable to the inspector without causing the bakery
this practice. He now plans to buy new equipment that being fined?
guarantees the thickness of the tortilla within 0.005
inches. If the ideal thickness of the tortilla is 0.04
inches, what thickness of tortillas will be guaranteed?

Writing Exercises
494. Write a graphical description of the absolute value 495. In your own words, explain how to solve the
of a number. absolute value inequality, |3x − 2| ≥ 4.

Self Check

ⓐ After completing the exercises, use this checklist to evaluate your mastery of the objectives of this section.

ⓑ What does this checklist tell you about your mastery of this section? What steps will you take to improve?
210 Chapter 2 Solving Linear Equations

CHAPTER 2 REVIEW

KEY TERMS
compound inequality A compound inequality is made up of two inequalities connected by the word “and” or the word
“or.”
conditional equation An equation that is true for one or more values of the variable and false for all other values of the
variable is a conditional equation.
contradiction An equation that is false for all values of the variable is called a contradiction. A contradiction has no
solution.
identity An equation that is true for any value of the variable is called an Identity. The solution of an identity is all real
numbers.
linear equation A linear equation is an equation in one variable that can be written, where a and b are real numbers and
a ≠ 0, as ax + b = 0.
solution of an equation A solution of an equation is a value of a variable that makes a true statement when substituted
into the equation.

KEY CONCEPTS
2.1 Use a General Strategy to Solve Linear Equations
• How to determine whether a number is a solution to an equation
Step 1. Substitute the number in for the variable in the equation.
Step 2. Simplify the expressions on both sides of the equation.
Step 3. Determine whether the resulting equation is true.
If it is true, the number is a solution.
If it is not true, the number is not a solution.
• How to Solve Linear Equations Using a General Strategy
Step 1. Simplify each side of the equation as much as possible.
Use the Distributive Property to remove any parentheses.
Combine like terms.
Step 2. Collect all the variable terms on one side of the equation.
Use the Addition or Subtraction Property of Equality.
Step 3. Collect all the constant terms on the other side of the equation.
Use the Addition or Subtraction Property of Equality.
Step 4. Make the coefficient of the variable term equal to 1.
Use the Multiplication or Division Property of Equality.
State the solution to the equation.
Step 5. Check the solution.
Substitute the solution into the original equation to make sure the result is a true statement.
• How to Solve Equations with Fraction or Decimal Coefficients
Step 1. Find the least common denominator (LCD) of all the fractions and decimals (in fraction form) in the
equation.
Step 2. Multiply both sides of the equation by that LCD. This clears the fractions and decimals.
Step 3. Solve using the General Strategy for Solving Linear Equations.

2.2 Use a Problem Solving Strategy


• How To Use a Problem Solving Strategy for Word Problems
Step 1. Read the problem. Make sure all the words and ideas are understood.
Step 2. Identify what you are looking for.
Step 3. Name what you are looking for. Choose a variable to represent that quantity.
Step 4. Translate into an equation. It may be helpful to restate the problem in one sentence with all the important
information. Then, translate the English sentence into an algebra equation.

This OpenStax book is available for free at http://cnx.org/content/col12119/1.5


Chapter 2 Solving Linear Equations 211

Step 5. Solve the equation using proper algebra techniques.

Step 6. Check the answer in the problem to make sure it makes sense.
Step 7. Answer the question with a complete sentence.
• How To Find Percent Change
Step 1. Find the amount of change
change = new amount − original amount
Step 2. Find what percent the amount of change is of the original amount.
change is what percent of the original amount?
• Discount
amount of discount = discount rate · original price
sale price = original amount − discount
• Mark-up
amount of mark-up = mark-up rate · original cost
list price = original cost + mark up
• Simple Interest
If an amount of money, P, called the principal, is invested or borrowed for a period of t years at an annual interest
rate r, the amount of interest, I, earned or paid is:
I = interest
I = Prt where P = principal
r = rate
t = time

2.3 Solve a Formula for a Specific Variable


• How To Solve Geometry Applications
Step 1. Read the problem and make sure all the words and ideas are understood.
Step 2. Identify what you are looking for.
Step 3. Name what you are looking for by choosing a variable to represent it. Draw the figure and label it with the
given information.
Step 4. Translate into an equation by writing the appropriate formula or model for the situation. Substitute in the
given information.
Step 5. Solve the equation using good algebra techniques.
Step 6. Check the answer in the problem and make sure it makes sense.
Step 7. Answer the question with a complete sentence.
• The Pythagorean Theorem
◦ In any right triangle, where a and b are the lengths of the legs, and c is the length of the hypotenuse, the
sum of the squares of the lengths of the two legs equals the square of the length of the hypotenuse.

2.4 Solve Mixture and Uniform Motion Applications


• Total Value of Coins
For the same type of coin, the total value of a number of coins is found by using the model
number · value = total value
◦ number is the number of coins
212 Chapter 2 Solving Linear Equations

◦ value is the value of each coin


◦ total value is the total value of all the coins

• How to solve coin word problems.


Step 1. Read the problem. Make sure all the words and ideas are understood.
Determine the types of coins involved.
Create a table to organize the information.
Label the columns “type,” “number,” “value,” “total value.”
List the types of coins.
Write in the value of each type of coin.
Write in the total value of all the coins.

Step 2. Identify what you are looking for.


Step 3. Name what you are looking for. Choose a variable to represent that quantity.
Use variable expressions to represent the number of each type of coin and write them in the table.
Multiply the number times the value to get the total value of each type of coin.
Step 4. Translate into an equation.
It may be helpful to restate the problem in one sentence with all the important information. Then,
translate the sentence into an equation.
Write the equation by adding the total values of all the types of coins.
Step 5. Solve the equation using good algebra techniques.
Step 6. Check the answer in the problem and make sure it makes sense.

Step 7. Answer the question with a complete sentence.


• How To Solve a Uniform Motion Application
Step 1. Read the problem. Make sure all the words and ideas are understood.
Draw a diagram to illustrate what it happening.
Create a table to organize the information.
Label the columns rate, time, distance.
List the two scenarios.
Write in the information you know.

Step 2. Identify what you are looking for.


Step 3. Name what you are looking for. Choose a variable to represent that quantity.
Complete the chart.
Use variable expressions to represent that quantity in each row.
Multiply the rate times the time to get the distance.
Step 4. Translate into an equation.
Restate the problem in one sentence with all the important information.
Then, translate the sentence into an equation.
Step 5. Solve the equation using good algebra techniques.
Step 6. Check the answer in the problem and make sure it makes sense.
Step 7. Answer the question with a complete sentence.

This OpenStax book is available for free at http://cnx.org/content/col12119/1.5


Chapter 2 Solving Linear Equations 213

2.5 Solve Linear Inequalities


• Inequalities, Number Lines, and Interval Notation
x>a x≥a x<a x≤a

• Linear Inequality
◦ A linear inequality is an inequality in one variable that can be written in one of the following forms where
a, b, and c are real numbers and a ≠ 0 :

ax + b < c, ax + b ≤ c, ax + b > c, ax + b ≥ c.
• Addition and Subtraction Property of Inequality
◦ For any numbers a, b, and c, if a < b, then
a+c<b+c a−c<b−c
a+c>b+c a−c>b−c
◦ We can add or subtract the same quantity from both sides of an inequality and still keep the inequality.
• Multiplication and Division Property of Inequality
◦ For any numbers a, b, and c,
multiply or divide by a positive
if a < b and c > 0, then ac < bc and ac < bc .
if a > b and c > 0, then ac > bc and ac > bc .

multiply or divide by a negative


if a < b and c < 0, then ac > bc and ac > bc .
if a > b and c < 0, then ac < bc and ac < bc .

• Phrases that indicate inequalities

> ≥ < ≤

is greater than is greater than or equal to is less than is less than or equal to

is more than is at least is smaller than is at most

is larger than is no less than has fewer than is no more than

exceeds is the minimum is lower than is the maximum

2.6 Solve Compound Inequalities


• How to solve a compound inequality with “and”
Step 1. Solve each inequality.
Step 2. Graph each solution. Then graph the numbers that make both inequalities true. This graph shows the
solution to the compound inequality.
Step 3. Write the solution in interval notation.
• Double Inequality
214 Chapter 2 Solving Linear Equations

◦ A double inequality is a compound inequality such as a< x < b . It is equivalent to a < x and x < b.
a< x<b is equivalent to a< x and x<b
a≤ x≤b is equivalent to a≤ x and x≤b
Other forms:
a> x>b is equivalent to a> x and x>b
a≥ x≥b is equivalent to a≥ x and x≥b
• How to solve a compound inequality with “or”
Step 1. Solve each inequality.
Step 2. Graph each solution. Then graph the numbers that make either inequality true.
Step 3. Write the solution in interval notation.

2.7 Solve Absolute Value Inequalities


• Absolute Value
The absolute value of a number is its distance from 0 on the number line.
The absolute value of a number n is written as |n| and |n| ≥ 0 for all numbers.
Absolute values are always greater than or equal to zero.
• Absolute Value Equations
For any algebraic expression, u, and any positive real number, a,
if |u| = a
then u = −a or u = a
Remember that an absolute value cannot be a negative number.
• How to Solve Absolute Value Equations
Step 1. Isolate the absolute value expression.
Step 2. Write the equivalent equations.
Step 3. Solve each equation.
Step 4. Check each solution.
• Equations with Two Absolute Values
For any algebraic expressions, u and v,
if |u| = |v|
then u = −v or u = v
• Absolute Value Inequalities with < or ≤
For any algebraic expression, u, and any positive real number, a,
if |u| < a, then −a < u < a
if |u| ≤ a, then −a ≤ u ≤ a
• How To Solve Absolute Value Inequalities with < or ≤
Step 1. Isolate the absolute value expression.
Step 2. Write the equivalent compound inequality.
|u| < a is equivalent to −a < u < a
|u| ≤ a is equivalent to −a ≤ u ≤ a
Step 3. Solve the compound inequality.
Step 4. Graph the solution
Step 5. Write the solution using interval notation
• Absolute Value Inequalities with > or ≥
For any algebraic expression, u, and any positive real number, a,
if |u| > a, then u < −a or u > a
if |u| ≥ a, then u ≤ −a or u ≥ a
• How To Solve Absolute Value Inequalities with > or ≥
Step 1. Isolate the absolute value expression.

This OpenStax book is available for free at http://cnx.org/content/col12119/1.5


Chapter 2 Solving Linear Equations 215

Step 2. Write the equivalent compound inequality.


|u| > a is equivalent to u < −a or u > a
|u| ≥ a is equivalent to u ≤ −a or u ≥ a
Step 3. Solve the compound inequality.
Step 4. Graph the solution
Step 5. Write the solution using interval notation

REVIEW EXERCISES
2.1 Use a General Strategy to Solve Linear Equations
Solve Equations Using the General Strategy for Solving Linear Equations
In the following exercises, determine whether each number is a solution to the equation.

496. 10x − 1 = 5x, x = 1 497. −12n + 5 = 8n, n = − 5


5 4

In the following exercises, solve each linear equation.


498. 6(x + 6) = 24 499. −(s + 4) = 18

500. 23 − 3⎛⎝y − 7⎞⎠ = 8 501. 1 (6m + 21) = m − 7


3

502. 4⎛⎝3.5y + 0.25⎞⎠ = 365 503. 0.25⎛⎝q − 8⎞⎠ = 0.1⎛⎝q + 7⎞⎠

504. 8(r − 2) = 6(r + 10) 505. 5 + 7(2 − 5x) = 2(9x + 1) − (13x − 57)

506. (9n + 5) − (3n − 7) = 20 − (4n − 2) 507. 2⎡⎣−16 + 5(8k − 6)⎤⎦ = 8(3 − 4k) − 32

Classify Equations
In the following exercises, classify each equation as a conditional equation, an identity, or a contradiction and then state the
solution.
508. 17y − 3⎛⎝4 − 2y⎞⎠ = 11⎛⎝y − 1⎞⎠ + 12y − 1 509. 9u + 32 = 15(u − 4) − 3(2u + 21)

510. −8(7m + 4) = −6(8m + 9)

Solve Equations with Fraction or Decimal Coefficients


In the following exercises, solve each equation.

511. 2n − 1 = 7 512. 3a − 1 = 1a + 5
5 10 10 4 3 2 6

513. 1 (k + 3) = 1 (k + 16) 5y − 1 −8y + 4


2 3 514. +4=
3 6

515. 0.8x − 0.3 = 0.7x + 0.2 516. 0.10d + 0.05(d − 4) = 2.05


216 Chapter 2 Solving Linear Equations

2.2 Use a Problem-Solving Strategy


Use a Problem Solving Strategy for Word Problems
In the following exercises, solve using the problem solving strategy for word problems.
517. Three-fourths of the people 518. There are nine saxophone
at a concert are children. If there players in the band. The number of
are 87 children, what is the total saxophone players is one less than
number of people at the concert? twice the number of tuba players.
Find the number of tuba players.

Solve Number Word Problems


In the following exercises, solve each number word problem.
519. The sum of a number and 520. One number is nine less than 521. One number is two more
three is forty-one. Find the another. Their sum is negative than four times another. Their sum
number. twenty-seven. Find the numbers. is negative thirteen. Find the
numbers.

522. The sum of two consecutive 523. Find three consecutive even 524. Find three consecutive odd
integers is −135. Find the integers whose sum is 234. integers whose sum is 51.
numbers.

525. Koji has $5,502 in his savings


account. This is $30 less than six
times the amount in his checking
account. How much money does
Koji have in his checking account?

Solve Percent Applications


In the following exercises, translate and solve.
526. What number is 67% of 250? 527. 12.5% of what number is 20? 528. What percent of 125 is 150?

In the following exercises, solve.


529. The bill for Dino’s lunch was 530. Dolores bought a crib on sale 531. Jaden earns $2,680 per
$19.45. He wanted to leave 20% of for $350. The sale price was 40% month. He pays $938 a month for
the total bill as a tip. How much of the original price. What was the rent. What percent of his monthly
should the tip be? original price of the crib? pay goes to rent?

532. Angel received a raise in his 533. Rowena’s monthly gasoline 534. Emmett bought a pair of
annual salary from $55,400 to bill dropped from $83.75 last shoes on sale at 40% off from an
$56,785. Find the percent change. month to $56.95 this month. Find
the percent change.
original price of $138. Find ⓐ the
amount of discount and ⓑ the sale
price.

535. Lacey bought a pair of boots 536. Nga and Lauren bought a
on sale for $95. The original price chest at a flea market for $50. They
of the boots was $200. Find ⓐ the re-finished it and then added a
350% mark-up. Find ⓐ the amount
amount of discount and ⓑ the
discount rate. (Round to the of the mark-up and ⓑ the list
nearest tenth of a percent, if price.
needed.)

This OpenStax book is available for free at http://cnx.org/content/col12119/1.5


Chapter 2 Solving Linear Equations 217

Solve Simple Interest Applications


In the following exercises, solve.
537. Winston deposited $3,294 in 538. Moira borrowed $4,500 from 539. Jaime’s refrigerator loan
a bank account with interest rate her grandfather to pay for her first statement said he would pay
2.6% How much interest was year of college. Three years later, $1,026 in interest for a four-year
earned in five years? she repaid the $4,500 plus $243 loan at 13.5%. How much did Jaime
interest. What was the rate of borrow to buy the refrigerator?
interest?

2.3 Solve a formula for a Specific Variable


Solve a Formula for a Specific Variable
In the following exercises, solve the formula for the specified variable.
540. Solve the formula 541. Solve the formula 542. Solve the formula
V = LWH for L. A = 1 d 1 d 2 for d 2. h = 48t + 1 at 2 for t.
2 2

543. Solve the formula


4x − 3y = 12 for y.

Use Formulas to Solve Geometry Applications


In the following exercises, solve using a geometry formula.
544. What is the height of a 545. The measure of the smallest 546. The perimeter of a triangle is
triangle with area 67.5 square angle in a right triangle is 45° less 97 feet. One side of the triangle is
meters and base 9 meters? than the measure of the next eleven feet more than the smallest
larger angle. Find the measures of side. The third side is six feet more
all three angles. than twice the smallest side. Find
the lengths of all sides.

547. Find the length of the 548. Find the length of the missing 549. Sergio needs to attach a wire
hypotenuse. side. Round to the nearest tenth, if to hold the antenna to the roof of
necessary. his house, as shown in the figure.
The antenna is eight feet tall and
Sergio has 10 feet of wire. How far
from the base of the antenna can
he attach the wire? Approximate to
the nearest tenth, if necessary.

550. Seong is building shelving in 551. The length of a rectangle is 552. The width of a rectangle is
his garage. The shelves are 36 12 cm more than the width. The three more than twice the length.
inches wide and 15 inches tall. He perimeter is 74 cm. Find the length The perimeter is 96 inches. Find the
wants to put a diagonal brace and the width. length and the width.
across the back to stabilize the
shelves, as shown. How long
should the brace be?
218 Chapter 2 Solving Linear Equations

553. The perimeter of a triangle


is 35 feet. One side of the triangle
is five feet longer than the second
side. The third side is three feet
longer than the second side. Find
the length of each side.

2.4 Solve Mixture and Uniform Motion Applications


Solve Coin Word Problems
In the following exercises, solve.
554. Paulette has $140 in $5 and 555. Lenny has $3.69 in pennies,
$10 bills. The number of $10 bills dimes, and quarters. The number
is one less than twice the number of pennies is three more than the
of $5 bills. How many of each does number of dimes. The number of
she have? quarters is twice the number of
dimes. How many of each coin
does he have?

Solve Ticket and Stamp Word Problems


In the following exercises, solve each ticket or stamp word problem.
556. Tickets for a basketball game 557. 125 tickets were sold for the 558. Yumi spent $34.15 buying
cost $2 for students and $5 for jazz band concert for a total of stamps. The number of $0.56
adults. The number of students $1,022. Student tickets cost $6 each stamps she bought was 10 less
was three less than 10 times the and general admission tickets cost than four times the number of
number of adults. The total $10 each. How many of each kind $0.41 stamps. How many of each
amount of money from ticket sales of ticket were sold? did she buy?
was $619. How many of each ticket
were sold?

Solve Mixture Word Problems


In the following exercises, solve.
559. Marquese is making 10 560. Amber wants to put tiles on 561. Enrique borrowed $23,500 to
pounds of trail mix from raisins the backsplash of her kitchen buy a car. He pays his uncle 2%
and nuts. Raisins cost $3.45 per counters. She will need 36 square interest on the $4,500 he borrowed
pound and nuts cost $7.95 per feet of tile. She will use basic tiles from him, and he pays the bank
pound. How many pounds of that cost $8 per square foot and 11.5% interest on the rest. What
raisins and how many pounds of decorator tiles that cost $20 per average interest rate does he pay
nuts should Marquese use for the square foot. How many square feet on the total $23,500? (Round your
trail mix to cost him $6.96 per of each tile should she use so that answer to the nearest tenth of a
pound? the overall cost of the backsplash percent.)
will be $10 per square foot?

Solve Uniform Motion Applications


In the following exercises, solve.
562. When Gabe drives from 563. Louellen and Tracy met at a 564. Two busses leave Amarillo at
Sacramento to Redding it takes restaurant on the road between the same time. The Albuquerque
him 2.2 hours. It takes Elsa two Chicago and Nashville. Louellen bus heads west on the I-40 at a
hours to drive the same distance. had left Chicago and drove 3.2 speed of 72 miles per hour, and the
Elsa’s speed is seven miles per hours towards Nashville. Tracy had Oklahoma City bus heads east on
hour faster than Gabe’s speed. left Nashville and drove 4 hours the I-40 at a speed of 78 miles per
Find Gabe’s speed and Elsa’s towards Chicago, at a speed one hour. How many hours will it take
speed. mile per hour faster than them to be 375 miles apart?
Louellen’s speed. The distance
between Chicago and Nashville is
472 miles. Find Louellen’s speed
and Tracy’s speed.

This OpenStax book is available for free at http://cnx.org/content/col12119/1.5


Chapter 2 Solving Linear Equations 219

565. Kyle rowed his boat upstream 566. At 6:30, Devon left her house 567. Anthony drove from New
for 50 minutes. It took him 30 and rode her bike on the flat road York City to Baltimore, which is a
minutes to row back downstream. until 7:30. Then she started riding distance of 192 miles. He left at
His speed going upstream is two uphill and rode until 8:00. She rode 3:45 and had heavy traffic until
miles per hour slower than his a total of 15 miles. Her speed on 5:30. Traffic was light for the rest
speed going downstream. Find the flat road was three miles per of the drive, and he arrived at 7:30.
Kyle’s upstream and downstream hour faster than her speed going His speed in light traffic was four
speeds. uphill. Find Devon’s speed on the miles per hour more than twice his
flat road and riding uphill. speed in heavy traffic. Find
Anthony’s driving speed in heavy
traffic and light traffic.

2.5 Solve Linear Inequalities


Graph Inequalities on the Number Line
In the following exercises, graph the inequality on the number line and write in interval notation.
568. x < −1 569. x ≥ −2.5 570. x≤5
4

571. x>2 572. −2 < x < 0 573. −5 ≤ x < −3

574. 0 ≤ x ≤ 3.5

Solve Linear Inequalities


In the following exercises, solve each inequality, graph the solution on the number line, and write the solution in interval notation.
575. n − 12 ≤ 23 576. a+ 2 ≥ 7 577. 9x > 54
3 12

q 579. 6p > 15p − 30 580. 9h − 7(h − 1) ≤ 4h − 23


578. ≥ −24
−2

581.
582. 3a − 1 a > 5 a + 3
5n − 15(4 − n) < 10(n − 6) + 10n 8 12 12 4

Translate Words to an Inequality and Solve


In the following exercises, translate and solve. Then write the solution in interval notation and graph on the number line.
583. Five more than z is at most 584. Three less than c is at least 585. Nine times n exceeds 42.
19. 360.

586. Negative two times a is no


more than eight.

Solve Applications with Linear Inequalities


In the following exercises, solve.
587. Julianne has a weekly food 588. Rogelio paints watercolors. 589. Briana has been offered a
budget of $231 for her family. If He got a $100 gift card to the art sales job in another city. The offer
she plans to budget the same supply store and wants to use it was for $42,500 plus 8% of her total
amount for each of the seven days to buy 12″ × 16″ canvases. Each sales. In order to make it worth the
of the week, what is the maximum canvas costs $10.99. What is the move, Briana needs to have an
amount she can spend on food maximum number of canvases he annual salary of at least $66,500.
each day? can buy with his gift card? What would her total sales need to
be for her to move?
220 Chapter 2 Solving Linear Equations

590. Renee’s car costs her $195 591. Costa is an accountant. 592. Jenna is planning a five-day
per month plus $0.09 per mile. During tax season, he charges resort vacation with three of her
How many miles can Renee drive $125 to do a simple tax return. His friends. It will cost her $279 for
so that her monthly car expenses expenses for buying software, airfare, $300 for food and
are no more than $250? renting an office, and advertising entertainment, and $65 per day for
are $6,000. How many tax returns her share of the hotel. She has
must he do if he wants to make a $550 saved towards her vacation
profit of at least $8,000? and can earn $25 per hour as an
assistant in her uncle’s
photography studio. How many
hours must she work in order to
have enough money for her
vacation?

2.6 Solve Compound Inequalities


Solve Compound Inequalities with “and”
In each of the following exercises, solve each inequality, graph the solution, and write the solution in interval notation.
593. x ≤ 5 and x > −3 594. 4x − 2 ≤ 4 and 595. 5(3x − 2) ≤ 5 and
7x − 1 > −8 4(x + 2) < 3

596. 3 (x − 8) ≤ 3 and 597. 3 x − 5 ≥ −2 and 598. −5 ≤ 4x − 1 < 7


4 4
1 (x − 5) ≤ 3 −3(x + 1) ≥ 6
5

Solve Compound Inequalities with “or”


In the following exercises, solve each inequality, graph the solution on the number line, and write the solution in interval notation.
599. 5 − 2x ≤ −1 or 600. 3(2x − 3) < −5 or 601. 3 x − 2 > 4 or 4(2 − x) > 0
6 + 3x ≤ 4 4x − 1 > 3 4

602. 2(x + 3) ≥ 0 or 603. 1 x − 3 ≤ 4 or


3(x + 4) ≤ 6 2
1 (x − 6) ≥ −2
3

Solve Applications with Compound Inequalities


In the following exercises, solve.
604. Liam is playing a number 605. Elouise is creating a
game with his sister Audry. Liam rectangular garden in her back
is thinking of a number and wants yard. The length of the garden is 12
Audry to guess it. Five more than feet. The perimeter of the garden
three times her number is between must be at least 36 feet and no
2 and 32. Write a compound more than 48 feet. Use a
inequality that shows the range of compound inequality to find the
numbers that Liam might be range of values for the width of the
thinking of. garden.

2.7 Solve Absolute Value Inequalities


Solve Absolute Value Equations
In the following exercises, solve.
606. | x| = 8 607. |y| = −14 608. |z| = 0

609. |3x − 4| + 5 = 7 610. 4|x − 1| + 2 = 10 611. −2|x − 3| + 8 = −4

This OpenStax book is available for free at http://cnx.org/content/col12119/1.5


Chapter 2 Solving Linear Equations 221

612.
|12 x + 5| + 4 = 1 613. |6x − 5| = |2x + 3|

Solve Absolute Value Inequalities with “less than”


In the following exercises, solve each inequality. Graph the solution and write the solution in interval notation.
614. | x| ≤ 8 615. |2x − 5| ≤ 3 616. |6x − 5| < 7

617. |5x + 1| ≤ −2

Solve Absolute Value Inequalities with “greater than”


In the following exercises, solve. Graph the solution and write the solution in interval notation.
618. | x| > 6 619. | x| ≥ 2 620. |x − 5| > −2

621. |x − 7| ≥ 1 622. 3|x| + 4 ≥ 1

Solve Applications with Absolute Value


In the following exercises, solve.
623. A craft beer brewer needs 624. At Fancy Grocery, the ideal
215,000 bottle per day. But this weight of a loaf of bread is 16
total can vary by as much as 5,000 ounces. By law, the actual weight
bottles. What is the maximum and can vary from the ideal by 1.5
minimum expected usage at the ounces. What range of weight will
bottling company? be acceptable to the inspector
without causing the bakery being
fined?
222 Chapter 2 Solving Linear Equations

PRACTICE TEST
In the following exercises, solve each equation.
625. −5(2x + 1) = 45 626. 627.
1 (12m + 28) = 6 + 2(3m + 1) 8(3a + 5) − 7(4a − 3) = 20 − 3a
4

628. 0.1d + 0.25(d + 8) = 4.1 629. 630.


14n − 3(4n + 5) = −9 + 2(n − 8) 3(3u + 2) + 4⎡⎣6 − 8(u − 1)⎤⎦ = 3(u − 2)

631. 3x − 2 = 1x + 5 632. |3x − 4| = 8 633. |2x − 1| = |4x + 3|


4 3 2 6

634. Solve the formula


x + 2y = 5 for y.

In the following exercises, graph the inequality on the number line and write in interval notation.
635. x ≥ −3.5 636. x < 11 637. −2 ≤ x < 5
4

In the following exercises, solve each inequality, graph the solution on the number line, and write the solution in interval notation.
638. 8k ≥ 5k − 120 639. 3c − 10(c − 2) < 5c + 16 640. 3 x − 5 ≥ −2 and
4
−3(x + 1) ≥ 6

641. 3(2x − 3) < −5 or 642. 1 x − 3 ≤ 4 or 643. |4x − 3| ≥ 5


4x − 1 > 3 2
1 (x − 6) ≥ −2
3

In the following exercises, translate to an equation or inequality and solve.


644. Four less than twice x is 16. 645. Find the length of the missing 646. One number is four more
side. than twice another. Their sum is
−47. Find the numbers.

647. The sum of two consecutive 648. Marcus bought a television 649. Bonita has $2.95 in dimes and
odd integers is −112. Find the on sale for $626.50 The original quarters in her pocket. If she has
numbers. price of the television was $895. five more dimes than quarters,
Find ⓐ the amount of discount and how many of each coin does she
have?
ⓑ the discount rate.
650. Kim is making eight gallons 651. The measure of one angle of 652. The length of a rectangle is
of punch from fruit juice and soda. a triangle is twice the measure of five feet more than four times the
The fruit juice costs $6.04 per the smallest angle. The measure of width. The perimeter is 60 feet.
gallon and the soda costs $4.28 per the third angle is three times the Find the dimensions of the
gallon. How much fruit juice and measure of the smallest angle. rectangle.
how much soda should she use so Find the measures of all three
that the punch costs $5.71 per angles.
gallon?

This OpenStax book is available for free at http://cnx.org/content/col12119/1.5


Chapter 2 Solving Linear Equations 223

653. Two planes leave Dallas at 654. Leon drove from his house in 655. Sara has a budget of $1,000
the same time. One heads east at Cincinnati to his sister’s house in for costumes for the 18 members
a speed of 428 miles per hour. The Cleveland, a distance of 252 miles. of her musical theater group. What
other plane heads west at a speed
It took him 4 1 hours. For the first is the maximum she can spend for
of 382 miles per hour. How many 2 each costume?
hours will it take them to be 2,025 half hour, he had heavy traffic, and
miles apart? the rest of the time his speed was
five miles per hour less than twice
his speed in heavy traffic. What was
his speed in heavy traffic?
224 Chapter 2 Solving Linear Equations

This OpenStax book is available for free at http://cnx.org/content/col12119/1.5


Chapter 3 Graphs and Functions 225

3 GRAPHS AND FUNCTIONS

Figure 3.1 This odd-looking headgear provides the user with a virtual world. (credit: fill/Pixabay)

Chapter Outline
3.1 Graph Linear Equations in Two Variables
3.2 Slope of a Line
3.3 Find the Equation of a Line
3.4 Graph Linear Inequalities in Two Variables
3.5 Relations and Functions
3.6 Graphs of Functions

Introduction
Imagine visiting a faraway city or even outer space from the comfort of your living room. It could be possible using virtual
reality. This technology creates realistic images that make you feel as if you are truly immersed in the scene and even
enable you to interact with them. It is being developed for fun applications, such as video games, but also for architects
to plan buildings, car companies to design prototypes, the military to train, and medical students to learn.
Developing virtual reality devices requires modeling the environment using graphs and mathematical relationships. In
this chapter, you will graph different relationships and learn ways to describe and analyze graphs.

3.1
Graph Linear Equations in Two Variables
Learning Objectives
By the end of this section, you will be able to:
Plot points in a rectangular coordinate system
Graph a linear equation by plotting points
Graph vertical and horizontal lines
Find the x- and y-intercepts
Graph a line using the intercepts

Be Prepared!

Before you get started, take this readiness quiz.


1. Evaluate 5x − 4 when x = −1.
If you missed this problem, review Example 1.6.
2. Evaluate 3x − 2y when x = 4, y = −3.
If you missed this problem, review Example 1.21.
226 Chapter 3 Graphs and Functions

3. Solve for y: 8 − 3y = 20.


If you missed this problem, review Example 2.2.

Plot Points on a Rectangular Coordinate System


Just like maps use a grid system to identify locations, a grid system is used in algebra to show a relationship between
two variables in a rectangular coordinate system. The rectangular coordinate system is also called the xy-plane or the
“coordinate plane.”
The rectangular coordinate system is formed by two intersecting number lines, one horizontal and one vertical. The
horizontal number line is called the x-axis. The vertical number line is called the y-axis. These axes divide a plane into four
regions, called quadrants. The quadrants are identified by Roman numerals, beginning on the upper right and proceeding
counterclockwise. See Figure 3.2.

Figure 3.2

In the rectangular coordinate system, every point is represented by an ordered pair. The first number in the ordered pair
is the x-coordinate of the point, and the second number is the y-coordinate of the point. The phrase “ordered pair” means
that the order is important.

Ordered Pair

An ordered pair, (x, y) gives the coordinates of a point in a rectangular coordinate system. The first number is the
x-coordinate. The second number is the y-coordinate.

What is the ordered pair of the point where the axes cross? At that point both coordinates are zero, so its ordered pair is
(0, 0). The point (0, 0) has a special name. It is called the origin.

The Origin

The point (0, 0) is called the origin. It is the point where the x-axis and y-axis intersect.

We use the coordinates to locate a point on the xy-plane. Let’s plot the point (1, 3) as an example. First, locate 1 on the
x = 1. Then, locate 3 on the y-axis and sketch a horizontal line through
x-axis and lightly sketch a vertical line through
y = 3. Now, find the point where these two lines meet—that is the point with coordinates (1, 3). See Figure 3.3.

This OpenStax book is available for free at http://cnx.org/content/col12119/1.5


Chapter 3 Graphs and Functions 227

Figure 3.3

Notice that the vertical line throughx = 1 and the horizontal line through y = 3 are not part of the graph. We just used
them to help us locate the point (1, 3).

When one of the coordinate is zero, the point lies on one of the axes. In Figure 3.4 the point (0, 4) is on the y-axis and
the point (−2, 0) is on the x-axis.

Figure 3.4

Points on the Axes

Points with a y-coordinate equal to 0 are on the x-axis, and have coordinates (a, 0).
Points with an x-coordinate equal to 0 are on the y-axis, and have coordinates (0, b).

EXAMPLE 3.1

Plot each point in the rectangular coordinate system and identify the quadrant in which the point is located:

ⓐ (−5, 4) ⓑ (−3, −4) ⓒ (2, −3) ⓓ (0, −1) ⓔ ⎛⎝3, 52 ⎞⎠.


Solution
The first number of the coordinate pair is the x-coordinate, and the second number is the y-coordinate. To plot each
point, sketch a vertical line through the x-coordinate and a horizontal line through the y-coordinate. Their intersection
is the point.
ⓐ Since x = −5, the point is to the left of the y-axis. Also, since y = 4, the point is above the x-axis. The point
(−5, 4) is in Quadrant II.
ⓑ Since x = −3, the point is to the left of the y-axis. Also, since y = −4, the point is below the x-axis. The point
228 Chapter 3 Graphs and Functions

(−3, −4) is in Quadrant III.


ⓒ Since x = 2, the point is to the right of the y-axis. Since y = −3, the point is below the x-axis. The point (2, −3)
is in Quadrant IV.
ⓓ Since x = 0, the point whose coordinates are(0, −1) is on the y-axis.
ⓔ Since x = 3, the point is to the right of the y-axis. Since y = 5 , the point is above the x-axis. (It may be helpful to
2
⎛ ⎞
write 5 as a mixed number or decimal.) The point ⎝3, 5 ⎠ is in Quadrant I.
2 2

TRY IT : : 3.1

Plot each point in a rectangular coordinate system and identify the quadrant in which the point is located:
ⓐ (−2, 1) ⓑ (−3, −1) ⓒ (4, −4) ⓓ (−4, 4) ⓔ ⎛⎝−4, 32 ⎞⎠

TRY IT : : 3.2

Plot each point in a rectangular coordinate system and identify the quadrant in which the point is located:
ⓐ (−4, 1) ⓑ (−2, 3) ⓒ (2, −5) ⓓ (−2, 5) ⓔ ⎛⎝−3, 52 ⎞⎠

The signs of the x-coordinate and y-coordinate affect the location of the points. You may have noticed some patterns as
you graphed the points in the previous example. We can summarize sign patterns of the quadrants in this way:

Quadrants

Quadrant I Quadrant II Quadrant III Quadrant IV


⎛ ⎞ ⎛ ⎞ ⎛ ⎞ ⎛ ⎞
⎝ x, y⎠ ⎝ x, y⎠ ⎝ x, y⎠ ⎝ x, y⎠


⎝ + , + ⎞⎠ ⎛
⎝ − , + ⎞⎠ ⎛
⎝ − , − ⎞⎠ ⎛
⎝ + , − ⎞⎠

This OpenStax book is available for free at http://cnx.org/content/col12119/1.5


Chapter 3 Graphs and Functions 229

Up to now, all the equations you have solved were equations with just one variable. In almost every case, when you solved
the equation you got exactly one solution. But equations can have more than one variable. Equations with two variables
may be of the form Ax + By = C. An equation of this form is called a linear equation in two variables.

Linear Equation

An equation of the form Ax + By = C, where A and B are not both zero, is called a linear equation in two variables.

Here is an example of a linear equation in two variables, x and y.

The equation y = −3x + 5 is also a linear equation. But it does not appear to be in the form Ax + By = C. We can use
the Addition Property of Equality and rewrite it in Ax + By = C form.
y = −3x + 5
Add to both sides. y + 3x = −3x + 5 + 3x
Simplify. y + 3x = 5
Use the Commutative Property to put it in
Ax + By = C form. 3x + y = 5
By rewriting y = −3x + 5 as 3x + y = 5, we can easily see that it is a linear equation in two variables because it is
of the form Ax + By = C. When an equation is in the form Ax + By = C, we say it is in standard form of a linear
equation.

Standard Form of Linear Equation

A linear equation is in standard form when it is written Ax + By = C.

Most people prefer to have A, B, and C be integers and A ≥ 0 when writing a linear equation in standard form, although
it is not strictly necessary.
Linear equations have infinitely many solutions. For every number that is substituted for x there is a corresponding y
value. This pair of values is a solution to the linear equation and is represented by the ordered pair (x, y). When we
substitute these values of x and y into the equation, the result is a true statement, because the value on the left side is
equal to the value on the right side.

Solution of a Linear Equation in Two Variables

An ordered pair (x, y) is a solution of the linear equation Ax + By = C, if the equation is a true statement when
the x- and y-values of the ordered pair are substituted into the equation.
230 Chapter 3 Graphs and Functions

Linear equations have infinitely many solutions. We can plot these solutions in the rectangular coordinate system. The
points will line up perfectly in a straight line. We connect the points with a straight line to get the graph of the equation.
We put arrows on the ends of each side of the line to indicate that the line continues in both directions.
A graph is a visual representation of all the solutions of the equation. It is an example of the saying, “A picture is worth
a thousand words.” The line shows you all the solutions to that equation. Every point on the line is a solution of the
equation. And, every solution of this equation is on this line. This line is called the graph of the equation. Points not on the
line are not solutions!

Graph of a Linear Equation

The graph of a linear equation Ax + By = C is a straight line.


• Every point on the line is a solution of the equation.
• Every solution of this equation is a point on this line.

EXAMPLE 3.2

The graph of y = 2x − 3 is shown.

For each ordered pair, decide:

ⓐ Is the ordered pair a solution to the equation?


ⓑ Is the point on the line?
A: (0, −3) B: (3, 3) C: (2, −3) D: (−1, −5)
Solution
Substitute the x- and y-values into the equation to check if the ordered pair is a solution to the equation.

ⓑ Plot the points (0, −3), (3, 3), (2, −3), and (−1, −5).

This OpenStax book is available for free at http://cnx.org/content/col12119/1.5


Chapter 3 Graphs and Functions 231

The points (0, 3), (3, −3), and (−1, −5) are on the line y = 2x − 3, and the point (2, −3) is not on the line.
The points that are solutions to y = 2x − 3 are on the line, but the point that is not a solution is not on the line.

TRY IT : : 3.3 Use graph of y = 3x − 1. For each ordered pair, decide:

ⓐ Is the ordered pair a solution to the equation?


ⓑ Is the point on the line?
A (0, −1) B (2, 5)
232 Chapter 3 Graphs and Functions

TRY IT : : 3.4 Use graph of y = 3x − 1. For each ordered pair, decide:

ⓐ Is the ordered pair a solution to the equation?


ⓑ Is the point on the line?
A (3, −1) B (−1, −4)

Graph a Linear Equation by Plotting Points


There are several methods that can be used to graph a linear equation. The first method we will use is called plotting
points, or the Point-Plotting Method. We find three points whose coordinates are solutions to the equation and then plot
them in a rectangular coordinate system. By connecting these points in a line, we have the graph of the linear equation.

EXAMPLE 3.3 HOW TO GRAPH A LINEAR EQUATION BY PLOTTING POINTS

Graph the equation y = 2x + 1 by plotting points.

This OpenStax book is available for free at http://cnx.org/content/col12119/1.5


Chapter 3 Graphs and Functions 233

Solution
234 Chapter 3 Graphs and Functions

TRY IT : : 3.5 Graph the equation by plotting points: y = 2x − 3.

TRY IT : : 3.6 Graph the equation by plotting points: y = −2x + 4.

The steps to take when graphing a linear equation by plotting points are summarized here.

HOW TO : : GRAPH A LINEAR EQUATION BY PLOTTING POINTS.

Step 1. Find three points whose coordinates are solutions to the equation. Organize them in a table.
Step 2. Plot the points in a rectangular coordinate system. Check that the points line up. If they do not,
carefully check your work.
Step 3. Draw the line through the three points. Extend the line to fill the grid and put arrows on both
ends of the line.

It is true that it only takes two points to determine a line, but it is a good habit to use three points. If you only plot two
points and one of them is incorrect, you can still draw a line but it will not represent the solutions to the equation. It will
be the wrong line.
If you use three points, and one is incorrect, the points will not line up. This tells you something is wrong and you need to
check your work. Look at the difference between these illustrations.

When an equation includes a fraction as the coefficient of x , we can still substitute any numbers for x. But the arithmetic
is easier if we make “good” choices for the values of x. This way we will avoid fractional answers, which are hard to graph
precisely.

EXAMPLE 3.4

Graph the equation: y = 1 x + 3.


2
Solution
Find three points that are solutions to the equation. Since this equation has the fraction 1 as a coefficient of x, we will
2

This OpenStax book is available for free at http://cnx.org/content/col12119/1.5


Chapter 3 Graphs and Functions 235

choose values of x carefully. We will use zero as one choice and multiples of 2 for the other choices. Why are multiples
of two a good choice for values of x? By choosing multiples of 2 the multiplication by 1 simplifies to a whole number
2

The points are shown in Table 3.1.

y = 1x + 3
2

x y (x, y)

0 3 (0, 3)

2 4 (2, 4)

4 5 (4, 5)

Table 3.1

Plot the points, check that they line up, and draw the line.

TRY IT : : 3.7
Graph the equation: y = 1 x − 1.
3

TRY IT : : 3.8
Graph the equation: y = 1 x + 2.
4

Graph Vertical and Horizontal Lines


Some linear equations have only one variable. They may have just x and no y, or just y without an x. This changes how we
make a table of values to get the points to plot.
Let’s consider the equation x = −3. This equation has only one variable, x. The equation says that x is always equal to
236 Chapter 3 Graphs and Functions

−3, so its value does not depend on y. No matter what is the value of y, the value of x is always −3.
So to make a table of values, write −3 in for all the x-values. Then choose any values for y. Since x does not depend
on y, you can choose any numbers you like. But to fit the points on our coordinate graph, we’ll use 1, 2, and 3 for the
y-coordinates. See Table 3.2.

x = −3

x y (x, y)

−3 1 (−3, 1)

−3 2 (−3, 2)

−3 3 (−3, 3)

Table 3.2

Plot the points from the table and connect them with a straight line. Notice that we have graphed a vertical line.

What if the equation has y but no x? Let’s graph the equation y = 4. This time the y-value is a constant, so in this
equation, y does not depend on x. Fill in 4 for all the y’s in Table 3.3 and then choose any values for x. We’ll use 0, 2, and 4
for the x-coordinates.

y =4

x y (x, y)

0 4 (0, 4)

2 4 (2, 4)

4 4 (4, 4)

Table 3.3

In this figure, we have graphed a horizontal line passing through the y-axis at 4.

This OpenStax book is available for free at http://cnx.org/content/col12119/1.5


Chapter 3 Graphs and Functions 237

Vertical and Horizontal Lines

A vertical line is the graph of an equation of the form x = a.


The line passes through the x-axis at (a, 0).
A horizontal line is the graph of an equation of the form y = b.
The line passes through the y-axis at (0, b).

EXAMPLE 3.5

Graph: ⓐ x=2 ⓑ y = −1.


Solution
ⓐ The equation has only one variable, x, and x is always equal to 2. We create a table where x is always 2 and then put in
any values for y. The graph is a vertical line passing through the x-axis at 2.

x=2

x y (x, y)

2 1 (2, 1)

2 2 (2, 2)

2 3 (2, 3)
238 Chapter 3 Graphs and Functions

ⓑ Similarly, the equation y = −1 has only one variable, y. The value of y is constant. All the ordered pairs in the next
table have the same y-coordinate. The graph is a horizontal line passing through the y-axis at −1.

y = −1

x y (x, y)

0 −1 (0, −1)

3 −1 (3, −1)

−3 −1 (−3, −1)

TRY IT : : 3.9 Graph the equations: ⓐ x=5 ⓑ y = −4.

TRY IT : : 3.10 Graph the equations: ⓐ x = −2 ⓑ y = 3.

What is the difference between the equations y = 4x and y = 4 ?


The equation y = 4x has both x and y. The value of y depends on the value of x, so the y -coordinate changes according

This OpenStax book is available for free at http://cnx.org/content/col12119/1.5


Chapter 3 Graphs and Functions 239

to the value of x. The equation y = 4 has only one variable. The value of y is constant, it does not depend on the value of
x, so the y-coordinate is always 4.

Notice, in the graph, the equation y = 4x gives a slanted line, while y = 4 gives a horizontal line.

EXAMPLE 3.6

Graph y = −3x and y = −3 in the same rectangular coordinate system.


Solution
We notice that the first equation has the variable x, while the second does not. We make a table of points for each equation
and then graph the lines. The two graphs are shown.
240 Chapter 3 Graphs and Functions

TRY IT : : 3.11 Graph the equations in the same rectangular coordinate system: y = −4x and y = −4.

TRY IT : : 3.12 Graph the equations in the same rectangular coordinate system: y = 3 and y = 3x.

Find x- and y-intercepts


Every linear equation can be represented by a unique line that shows all the solutions of the equation. We have seen that
when graphing a line by plotting points, you can use any three solutions to graph. This means that two people graphing
the line might use different sets of three points.
At first glance, their two lines might not appear to be the same, since they would have different points labeled. But if all
the work was done correctly, the lines should be exactly the same. One way to recognize that they are indeed the same
line is to look at where the line crosses the x-axis and the y-axis. These points are called the intercepts of a line.

Intercepts of a Line

The points where a line crosses the x-axis and the y-axis are called the intercepts of the line.

Let’s look at the graphs of the lines.

First, notice where each of these lines crosses the x-axis. See Table 3.4.
Now, let’s look at the points where these lines cross the y-axis.

This OpenStax book is available for free at http://cnx.org/content/col12119/1.5


Chapter 3 Graphs and Functions 241

Figure The line crosses Ordered pair The line crosses Ordered pair
the x-axis at: for this point the y-axis at: for this point

Figure (a) 3 (3, 0) 6 (0, 6)

Figure (b) 4 (4, 0) −3 (0, −3)

Figure (c) 5 (5, 0) −5 (0, 5)

Figure (d) 0 (0, 0) 0 (0, 0)

General Figure a (a, 0) b (0, b)

Table 3.4

Do you see a pattern?


For each line, the y-coordinate of the point where the line crosses the x-axis is zero. The point where the line crosses the
x-axis has the form (a, 0) and is called the x-intercept of the line. The x-intercept occurs when y is zero.

In each line, the x-coordinate of the point where the line crosses the y-axis is zero. The point where the line crosses the
y-axis has the form (0, b) and is called the y-intercept of the line. The y-intercept occurs when x is zero.

x-intercept and y-intercept of a Line

The x-intercept is the point (a, 0) where the line crosses the x-axis.
The y-intercept is the point (0, b) where the line crosses the y-axis.

EXAMPLE 3.7

Find the x- and y-intercepts on each graph shown.


242 Chapter 3 Graphs and Functions

Solution
ⓐ The graph crosses the x-axis at the point (4, 0). The x-intercept is (4, 0).
The graph crosses the y-axis at the point (0, 2). The y-intercept is (0, 2).

ⓑ The graph crosses the x-axis at the point (2, 0). (2, 0).
The x-intercept is
The graph crosses the y-axis at the point (0, −6). The y-intercept is (0, −6).

ⓒ The graph crosses the x-axis at the point (−5, 0). The x-intercept is (−5, 0).
The graph crosses the y-axis at the point (0, −5). The y-intercept is (0, −5).

This OpenStax book is available for free at http://cnx.org/content/col12119/1.5


Chapter 3 Graphs and Functions 243

TRY IT : : 3.13 Find the x- and y-intercepts on the graph.

TRY IT : : 3.14 Find the x- and y-intercepts on the graph.

Recognizing that the x-intercept occurs when y is zero and that the y-intercept occurs when x is zero, gives us a method to
find the intercepts of a line from its equation. To find the x-intercept, let y = 0 and solve for x. To find the y-intercept, let
x = 0 and solve for y.

Find the x- and y-intercepts from the Equation of a Line

Use the equation of the line. To find:


• the x-intercept of the line, let y = 0 and solve for x.
• the y-intercept of the line, let x = 0 and solve for y.

EXAMPLE 3.8

Find the intercepts of 2x + y = 8.


Solution
We will let y = 0 to find the x-intercept, and let x = 0 to find the y-intercept. We will fill in a table, which reminds us
of what we need to find.
244 Chapter 3 Graphs and Functions

To find the x-intercept, let y = 0.

Let y = 0.

Simplify.

The x-intercept is: (4, 0)

To find the y-intercept, let x = 0.

Let x = 0.

Simplify.

The y-intercept is: (0, 8)

The intercepts are the points (4, 0) and (0, 8) as shown in the table.

2x + y = 8

x y

4 0

0 8

TRY IT : : 3.15 Find the intercepts: 3x + y = 12.

TRY IT : : 3.16 Find the intercepts: x + 4y = 8.

Graph a Line Using the Intercepts


To graph a linear equation by plotting points, you need to find three points whose coordinates are solutions to the
equation. You can use the x- and y- intercepts as two of your three points. Find the intercepts, and then find a third point
to ensure accuracy. Make sure the points line up—then draw the line. This method is often the quickest way to graph a
line.

EXAMPLE 3.9 HOW TO GRAPH A LINE USING THE INTERCEPTS

Graph – x + 2y = 6 using the intercepts.

This OpenStax book is available for free at http://cnx.org/content/col12119/1.5


Chapter 3 Graphs and Functions 245

Solution

TRY IT : : 3.17 Graph using the intercepts: x – 2y = 4.


246 Chapter 3 Graphs and Functions

TRY IT : : 3.18 Graph using the intercepts: – x + 3y = 6.

The steps to graph a linear equation using the intercepts are summarized here.

HOW TO : : GRAPH A LINEAR EQUATION USING THE INTERCEPTS.

Step 1. Find the x- and y-intercepts of the line.


◦ Let y = 0 and solve for x.
◦ Let x = 0 and solve for y.
Step 2. Find a third solution to the equation.
Step 3. Plot the three points and check that they line up.
Step 4. Draw the line.

EXAMPLE 3.10

Graph 4x − 3y = 12 using the intercepts.


Solution
Find the intercepts and a third point.

We list the points in the table and show the graph.

4x − 3y = 12

x y (x, y)

3 0 (3, 0)

0 −4 (0, −4)

6 4 (6, 4)

This OpenStax book is available for free at http://cnx.org/content/col12119/1.5


Chapter 3 Graphs and Functions 247

TRY IT : : 3.19 Graph using the intercepts: 5x − 2y = 10.

TRY IT : : 3.20 Graph using the intercepts: 3x − 4y = 12.

When the line passes through the origin, the x-intercept and the y-intercept are the same point.
248 Chapter 3 Graphs and Functions

EXAMPLE 3.11

Graph y = 5x using the intercepts.


Solution

This line has only one intercept. It is the point (0, 0).
To ensure accuracy, we need to plot three points. Since the x- and y-intercepts are the same point, we need two more
points to graph the line.

The resulting three points are summarized in the table.

y = 5x

x y (x, y)

0 0 (0, 0)

1 5 (1, 5)

−1 −5 (−1, −5)

Plot the three points, check that they line up, and draw the line.

TRY IT : : 3.21 Graph using the intercepts: y = 4x.

This OpenStax book is available for free at http://cnx.org/content/col12119/1.5


Chapter 3 Graphs and Functions 249

TRY IT : : 3.22 Graph the intercepts: y = −x.


250 Chapter 3 Graphs and Functions

3.1 EXERCISES
Practice Makes Perfect
Plot Points in a Rectangular Coordinate System
In the following exercises, plot each point in a rectangular coordinate system and identify the quadrant in which the point is
located.

1. ⓐ (−4, 2) ⓑ (−1, −2) ⓒ (3, −5) ⓓ (−3, 0) 2. ⓐ (−2, −3) ⓑ (3, −3) ⓒ (−4, 1) ⓓ (4, −1)
⎛5 ⎞ ⎛3 ⎞
ⓔ ⎝3 , 2⎠ ⓔ ⎝2 , 1⎠

3. ⓐ (3, −1) ⓑ (−3, 1) ⓒ (−2, 0) ⓓ (−4, −3) 4. ⓐ (−1, 1) ⓑ (−2, −1) ⓒ (2, 0) ⓓ (1, −4)
ⓔ ⎛⎝1, 14
5⎠

ⓔ ⎛ 7⎞
⎝3, 2 ⎠

In the following exercises, for each ordered pair, decide

ⓐ is the ordered pair a solution to the equation? ⓑ is the point on the line?
5. y = x + 2; 6. y = x − 4;
A: (0, 2); B: (1, 2); C: (−1, 1); D: (−3, −1). A: (0, −4); B: (3, −1); C: (2, 2); D: (1, −5).

This OpenStax book is available for free at http://cnx.org/content/col12119/1.5


Chapter 3 Graphs and Functions 251

7. y = 1 x − 3; 8. y = 1 x + 2;
2 3
A: (0, −3); B: (2, −2); C: (−2, −4); D: (4, 1) A: (0, 2); B: (3, 3); C: (−3, 2); D: (−6, 0).

Graph a Linear Equation by Plotting Points


In the following exercises, graph by plotting points.
9. y= x+2 10. y= x−3 11. y = 3x − 1

12. y = −2x + 2 13. y = −x − 3 14. y = −x − 2

15. y = 2x 16. y = −2x 17. y = 1x + 2


2

18. y = 1x − 1 19. y = 4x − 5 20. y = 3x − 3


3 3 2

21. y = − 2x + 1 22. y = − 4x − 1 23. y = − 3x + 2


5 5 2

24. y = − 5x + 4
3

Graph Vertical and Horizontal lines


In the following exercises, graph each equation.

25. ⓐ x=4 ⓑ y=3 26. ⓐ x=3 ⓑ y=1 27. ⓐ x = −2 ⓑ y = −5

28. ⓐ x = −5 ⓑ y = −2

In the following exercises, graph each pair of equations in the same rectangular coordinate system.
29. y = 2x and y = 2 30. y = 5x and y = 5 31. y = − 1 x and y = − 1
2 2

32. y = − 1 x and y = − 1
3 3
252 Chapter 3 Graphs and Functions

Find x- and y-Intercepts


In the following exercises, find the x- and y-intercepts on each graph.
33. 34.

35. 36.

In the following exercises, find the intercepts for each equation.


37. x−y=5 38. x − y = −4 39. 3x + y = 6

40. x − 2y = 8 41. 4x − y = 8 42. 5x − y = 5

43. 2x + 5y = 10 44. 3x − 2y = 12

Graph a Line Using the Intercepts


In the following exercises, graph using the intercepts.
45. −x + 4y = 8 46. x + 2y = 4 47. x + y = −3

48. x − y = −4 49. 4x + y = 4 50. 3x + y = 3

51. 3x − y = −6 52. 2x − y = −8 53. 2x + 4y = 12

54. 3x − 2y = 6 55. 2x − 5y = −20 56. 3x − 4y = −12

57. y = −2x 58. y = 5x 59. y=x

This OpenStax book is available for free at http://cnx.org/content/col12119/1.5


Chapter 3 Graphs and Functions 253

60. y = −x

Mixed Practice
In the following exercises, graph each equation.

61. y = 3x 62. y = − 2x 63. y = − 1x + 3


2 3 2

64. y = 1x − 2 65. 4x + y = 2 66. 5x + 2y = 10


4

67. y = −1 68. x=3

Writing Exercises
69. Explain how you would choose three x-values to 70. What is the difference between the equations of a
make a table to graph the line y = 1 x − 2.
vertical and a horizontal line?
5

71. Do you prefer to use the method of plotting points 72. Do you prefer to use the method of plotting points
or the method using the intercepts to graph the or the method using the intercepts to graph the
equation 4x + y = −4 ? Why? equation y = 2 x − 2 ? Why?
3

Self Check

ⓐ After completing the exercises, use this checklist to evaluate your mastery of the objectives of this section.

ⓑ If most of your checks were:


Confidently. Congratulations! You have achieved the objectives in this section. Reflect on the study skills you used so that you can
continue to use them. What did you do to become confident of your ability to do these things? Be specific.
With some help. This must be addressed quickly because topics you do not master become potholes in your road to success. In
math every topic builds upon previous work. It is important to make sure you have a strong foundation before you move on. Who
can you ask for help? Your fellow classmates and instructor are good resources. Is there a place on campus where math tutors
are available? Can your study skills be improved?
No, I don’t get it. This is a warning sign and you must address it. You should get help right away or you will quickly be
overwhelmed. See your instructor as soon as you can to discuss your situation. Together you can come up with a plan to get you
the help you need.
254 Chapter 3 Graphs and Functions

3.2
Slope of a Line
Learning Objectives
By the end of this section, you will be able to:
Find the slope of a line
Graph a line given a point and the slope
Graph a line using its slope and intercept
Choose the most convenient method to graph a line
Graph and interpret applications of slope–intercept
Use slopes to identify parallel and perpendicular lines

Be Prepared!

Before you get started, take this readiness quiz.


(1 – 4)
1. Simplify: .
(8 − 2)
If you missed this problem, review Example 1.30.

2. Divide: 0, 4.
4 0
If you missed this problem, review Example 1.49.

3. Simplify: 15 , −15 , −15 .


−3 3 −3
If you missed this problem, review Example 1.30.

Find the Slope of a Line


When you graph linear equations, you may notice that some lines tilt up as they go from left to right and some lines tilt
down. Some lines are very steep and some lines are flatter.
In mathematics, the measure of the steepness of a line is called the slope of the line.
The concept of slope has many applications in the real world. In construction the pitch of a roof, the slant of the plumbing
pipes, and the steepness of the stairs are all applications of slope. and as you ski or jog down a hill, you definitely
experience slope.
We can assign a numerical value to the slope of a line by finding the ratio of the rise and run. The rise is the amount the
vertical distance changes while the run measures the horizontal change, as shown in this illustration. Slope is a rate of
change. See Figure 3.5.

Figure 3.5

Slope of a Line

The slope of a line is m = rise


run .
The rise measures the vertical change and the run measures the horizontal change.

To find the slope of a line, we locate two points on the line whose coordinates are integers. Then we sketch a right triangle
where the two points are vertices and one side is horizontal and one side is vertical.
To find the slope of the line, we measure the distance along the vertical and horizontal sides of the triangle. The vertical
distance is called the rise and the horizontal distance is called the run,

This OpenStax book is available for free at http://cnx.org/content/col12119/1.5


Chapter 3 Graphs and Functions 255

HOW TO : : FIND THE SLOPE OF A LINE FROM ITS GRAPH USING m = rise
run .

Step 1. Locate two points on the line whose coordinates are integers.
Step 2. Starting with one point, sketch a right triangle, going from the first point to the second point.
Step 3. Count the rise and the run on the legs of the triangle.
Step 4.
Take the ratio of rise to run to find the slope: m = rise
run .

EXAMPLE 3.12

Find the slope of the line shown.

Solution

Locate two points on the graph whose (0, 5) and (3, 3)


coordinates are integers.

Starting at (0, 5), sketch a right triangle to


(3, 3) as shown in this graph.

Count the rise— since it goes down, it is negative. The rise is −2.

Count the run. The run is 3.

Use the slope formula. m = rise


run

Substitute the values of the rise and run. m = −2


3

Simplify. m= −2
3

The slope of the line is − 2 .


3
256 Chapter 3 Graphs and Functions

So y decreases by 2 units as x increases by 3 units.

TRY IT : : 3.23 Find the slope of the line shown.

TRY IT : : 3.24 Find the slope of the line shown.

How do we find the slope of horizontal and vertical lines? To find the slope of the horizontal line, y = 4, we could graph
the line, find two points on it, and count the rise and the run. Let’s see what happens when we do this, as shown in the
graph below.

What is the rise? The rise is 0.


What is the run? The run is 3.
What is the slope? m = rise
run
m= 0
3
m=0
The slope of the horizontal line y = 4 is 0.
Let’s also consider a vertical line, the line x = 3, as shown in the graph.

This OpenStax book is available for free at http://cnx.org/content/col12119/1.5


Chapter 3 Graphs and Functions 257

What is the rise? The rise is 2.


What is the run? The run is 0.

What is the slope? m = rise


run
m= 2
0
The slope is undefined since division by zero is undefined. So we say that the slope of the vertical line x = 3 is undefined.
All horizontal lines have slope 0. When the y-coordinates are the same, the rise is 0.
The slope of any vertical line is undefined. When the x-coordinates of a line are all the same, the run is 0.

Slope of a Horizontal and Vertical Line

The slope of a horizontal line, y = b, is 0.


The slope of a vertical line, x = a, is undefined.

EXAMPLE 3.13

Find the slope of each line: ⓐ x=8 ⓑ y = −5.


Solution
ⓐ x=8
This is a vertical line. Its slope is undefined.
ⓑ y = −5
This is a horizontal line. It has slope 0.

TRY IT : : 3.25 Find the slope of the line: x = −4.

TRY IT : : 3.26 Find the slope of the line: y = 7.

Quick Guide to the Slopes of Lines

Sometimes we’ll need to find the slope of a line between two points when we don’t have a graph to count out the rise and
the run. We could plot the points on grid paper, then count out the rise and the run, but as we’ll see, there is a way to find
the slope without graphing. Before we get to it, we need to introduce some algebraic notation.
We have seen that an ordered pair(x, y) gives the coordinates of a point. But when we work with slopes, we use two
points. How can the same symbol (x, y) be used to represent two different points? Mathematicians use subscripts to
distinguish the points.
258 Chapter 3 Graphs and Functions

(x 1, y 1) read “x sub 1, y sub 1”


(x 2, y 2) read “x sub 2, y sub 2”
We will use (x 1, y 1) to identify the first point and (x 2, y 2) to identify the second point.
If we had more than two points, we could use (x 3, y 3), (x 4, y 4), and so on.
Let’s see how the rise and run relate to the coordinates of the two points by taking another look at the slope of the line
between the points (2, 3) and (7, 6), as shown in this graph.

⎛x 1, y 1⎞ ⎛x 2, y 2⎞
Since we have two points, we will use subscript notation.
⎝ 2, 3 ⎠ ⎝ 7, 6 ⎠
m = rise
run
On the graph, we counted the rise of 3 and the run of 5. m= 3
5
Notice that the rise of 3 can be found by subtracting the
y-coordinates, 6 and 3, and the run of 5 can be found by
subtracting the x-coordinates 7 and 2.
We rewrite the rise and run by putting in the coordinates. m= 6−3
7−2

But 6 is y 2, the y-coordinate of the second point and 3 is y 1, the y-coordinate


y2 − y1
of the first point. So we can rewrite the slope using subscript notation. m=
7−2

Also 7 is the x-coordinate of the second point and 2 is the x-coordinate


y −y
of the first point. So again we rewrite the slope using subscript notation. m = x2 − x1
2 1

y −y
We’ve shown that m = x 2 − x 1 is really another version of m = rise
run . We can use this formula to find the slope of a line
2 1
when we have two points on the line.

Slope of a line between two points

The slope of the line between two points (x 1, y 1) and (x 2, y 2) is:


y −y
m = x2 − x1.
2 1

The slope is:


y of the second point minus y of the first point
over
x of the second point minus x of the first point.
EXAMPLE 3.14

This OpenStax book is available for free at http://cnx.org/content/col12119/1.5


Chapter 3 Graphs and Functions 259

Use the slope formula to find the slope of the line through the points (−2, −3) and (−7, 4).
Solution
⎛x 1, y 1 ⎞ ⎛x 2, y 2⎞
We’ll call (−2, −3) point #1 and (−7, 4) point #2. ⎝−2, −3⎠ ⎝−7, 4 ⎠
y −y
Use the slope formula. m = x2 − x1
2 1
Substitute the values.
y of the second point minus y of the first point
4 − (−3)
x of the second point minus x of the first point m=
−7 − (−2)
m= 7
−5
Simplify.
m= −7
5
Let’s verify this slope on the graph shown.

m = rise
run
m= 7
−5
m= −7
5

TRY IT : : 3.27

Use the slope formula to find the slope of the line through the pair of points: (−3, 4) and (2, −1).

TRY IT : : 3.28

Use the slope formula to find the slope of the line through the pair of points: (−2, 6) and (−3, −4).

Graph a Line Given a Point and the Slope


Up to now, in this chapter, we have graphed lines by plotting points, by using intercepts, and by recognizing horizontal
and vertical lines.
We can also graph a line when we know one point and the slope of the line. We will start by plotting the point and then
use the definition of slope to draw the graph of the line.

EXAMPLE 3.15 HOW TO GRAPH A LINE GIVEN A POINT AND THE SLOPE

Graph the line passing through the point (1, −1) whose slope is m = 3 .
4
260 Chapter 3 Graphs and Functions

Solution

You can check your work by finding a third point. Since the slope ism = 3 , it can also be written as m = −3 (negative
4 −4
divided by negative is positive!). Go back to (1, −1) and count out the rise, −3, and the run, −4.

TRY IT : : 3.29
Graph the line passing through the point (2, −2) with the slope m = 4 .
3

TRY IT : : 3.30
Graph the line passing through the point (−2, 3) with the slope m = 1 .
4

This OpenStax book is available for free at http://cnx.org/content/col12119/1.5


Chapter 3 Graphs and Functions 261

HOW TO : : GRAPH A LINE GIVEN A POINT AND THE SLOPE.

Step 1. Plot the given point.


Step 2.
Use the slope formula m = rise
run to identify the rise and the run.
Step 3. Starting at the given point, count out the rise and run to mark the second point.
Step 4. Connect the points with a line.

Graph a Line Using its Slope and Intercept


We have graphed linear equations by plotting points, using intercepts, recognizing horizontal and vertical lines, and using
one point and the slope of the line. Once we see how an equation in slope–intercept form and its graph are related, we’ll
have one more method we can use to graph lines.

See Figure 3.6. Let’s look at the graph of the equation y = 1 x + 3 and find its slope and y-intercept.
2

Figure 3.6

The red lines in the graph show us the rise is 1 and the run is 2. Substituting into the slope formula:

m = rise
run
m= 1
2

The y-intercept is (0, 3).


Look at the equation of this line.

Look at the slope and y-intercept.

When a linear equation is solved for y, the coefficient of the x term is the slope and the constant term is the y-coordinate
262 Chapter 3 Graphs and Functions

of the y-intercept. We say that the equation y = 1 x + 3 is in slope–intercept form. Sometimes the slope–intercept form
2
is called the “y-form.”

Slope Intercept Form of an Equation of a Line

The slope–intercept form of an equation of a line with slope m and y-intercept, (0, b) is y = mx + b.

Let’s practice finding the values of the slope and y-intercept from the equation of a line.

EXAMPLE 3.16

Identify the slope and y-intercept of the line from the equation:

ⓐ y = − 47 x − 2 ⓑ x + 3y = 9
Solution
ⓐ We compare our equation to the slope–intercept form of the equation.
Write the slope–intercept form of the equation of the line.

Write the equation of the line.

Identify the slope.

Identify the y-intercept.

ⓑ When an equation of a line is not given in slope–intercept form, our first step will be to solve the equation for y.
Solve for y. x + 3y = 9

Subtract x from each side.

Divide both sides by 3.

Simplify.

Write the slope–intercept form of the equation of the line.

Write the equation of the line.

Identify the slope.

Identify the y-intercept.

This OpenStax book is available for free at http://cnx.org/content/col12119/1.5


Chapter 3 Graphs and Functions 263

TRY IT : : 3.31 Identify the slope and y-intercept from the equation of the line.

ⓐ y = 25 x − 1 ⓑ x + 4y = 8

TRY IT : : 3.32 Identify the slope and y-intercept from the equation of the line.

ⓐ y = − 43 x + 1 ⓑ 3x + 2y = 12

We have graphed a line using the slope and a point. Now that we know how to find the slope and y-intercept of a line from
its equation, we can use the y-intercept as the point, and then count out the slope from there.

EXAMPLE 3.17

Graph the line of the equation y = −x + 4 using its slope and y-intercept.
Solution

y = mx + b

The equation is in slope–intercept form. y = −x + 4

Identify the slope and y-intercept. m = −1


y-intercept is (0, 4)

Plot the y-intercept. See the graph.

Identify the rise over the run. m = −1


1

Count out the rise and run to mark the second point. rise −1 , run 1

Draw the line as shown in the graph.

TRY IT : : 3.33 Graph the line of the equation y = −x − 3 using its slope and y-intercept.

TRY IT : : 3.34 Graph the line of the equation y = −x − 1 using its slope and y-intercept.

Now that we have graphed lines by using the slope and y-intercept, let’s summarize all the methods we have used to
graph lines.
264 Chapter 3 Graphs and Functions

Choose the Most Convenient Method to Graph a Line


Now that we have seen several methods we can use to graph lines, how do we know which method to use for a given
equation?
While we could plot points, use the slope–intercept form, or find the intercepts for any equation, if we recognize the most
convenient way to graph a certain type of equation, our work will be easier.
Generally, plotting points is not the most efficient way to graph a line. Let’s look for some patterns to help determine the
most convenient method to graph a line.
Here are five equations we graphed in this chapter, and the method we used to graph each of them.
Equation Method
#1 x=2 Vertical line
#2 y = −1 Horizontal line
#3 −x + 2y = 6 Intercepts
#4 4x − 3y = 12 Intercepts
#5 y = −x + 4 Slope–intercept
Equations #1 and #2 each have just one variable. Remember, in equations of this form the value of that one variable is
constant; it does not depend on the value of the other variable. Equations of this form have graphs that are vertical or
horizontal lines.
In equations #3 and #4, both x and y are on the same side of the equation. These two equations are of the form
Ax + By = C. We substituted y = 0 to find the x- intercept and x = 0 to find the y-intercept, and then found a third
point by choosing another value for x or y.
Equation #5 is written in slope–intercept form. After identifying the slope and y-intercept from the equation we used them
to graph the line.
This leads to the following strategy.

Strategy for Choosing the Most Convenient Method to Graph a Line

Consider the form of the equation.


• If it only has one variable, it is a vertical or horizontal line.
◦ x = a is a vertical line passing through the x-axis at a.
◦ y = b is a horizontal line passing through the y-axis at b.
• If y is isolated on one side of the equation, in the form y = mx + b, graph by using the slope and y-intercept.
◦ Identify the slope and y-intercept and then graph.
• If the equation is of the form Ax + By = C, find the intercepts.
◦ Find the x- and y-intercepts, a third point, and then graph.

EXAMPLE 3.18

Determine the most convenient method to graph each line:

This OpenStax book is available for free at http://cnx.org/content/col12119/1.5


Chapter 3 Graphs and Functions 265

ⓐ y = 5 ⓑ 4x − 5y = 20 ⓒ x = −3 ⓓ y = − 59 x + 8
Solution
ⓐ y=5
This equation has only one variable, y. Its graph is a horizontal line crossing the y-axis at 5.
ⓑ 4x − 5y = 20
This equation is of the form Ax + By = C. The easiest way to graph it will be to find the intercepts and one more point.
ⓒ x = −3
There is only one variable, x. The graph is a vertical line crossing the x-axis at −3.
ⓓ y = − 59 x + 8
Since this equation is in y = mx + b form, it will be easiest to graph this line by using the slope and y-intercepts.

TRY IT : : 3.35 Determine the most convenient method to graph each line:

ⓐ 3x + 2y = 12 ⓑ y = 4 ⓒ y = 15 x − 4 ⓓ x = −7.

TRY IT : : 3.36 Determine the most convenient method to graph each line:

ⓐ x = 6 ⓑ y = − 34 x + 1 ⓒ y = −8 ⓓ 4x − 3y = −1.

Graph and Interpret Applications of Slope–Intercept


Many real-world applications are modeled by linear equations. We will take a look at a few applications here so you can
see how equations written in slope–intercept form relate to real world situations.
Usually, when a linear equation models uses real-world data, different letters are used for the variables, instead of using
only x and y. The variable names remind us of what quantities are being measured.
Also, we often will need to extend the axes in our rectangular coordinate system to bigger positive and negative numbers
to accommodate the data in the application.

EXAMPLE 3.19

The equation F = 9 C + 32 is used to convert temperatures, C, on the Celsius scale to temperatures, F, on the Fahrenheit
5
scale.

ⓐ Find the Fahrenheit temperature for a Celsius temperature of 0.


ⓑ Find the Fahrenheit temperature for a Celsius temperature of 20.
ⓒ Interpret the slope and F-intercept of the equation.
ⓓ Graph the equation.
Solution

Find the Fahrenheit temperature for a Celsius temperature of 0. F = 9 C + 32
5
Find F when C = 0. F = 9 (0) + 32
5
Simplify. F = 32

266 Chapter 3 Graphs and Functions

Find the Fahrenheit temperature for a Celsius temperature of 20. F = 9 C + 32


5
Find F when C = 20. F = 9 (20) + 32
5
Simplify. F = 36 + 32
Simplify. F = 68

Interpret the slope and F-intercept of the equation.
Even though this equation uses F and C, it is still in slope–intercept form.

The slope, 9 , means that the temperature Fahrenheit (F) increases 9 degrees when the temperature Celsius (C)
5
increases 5 degrees.
The F-intercept means that when the temperature is 0° on the Celsius scale, it is 32° on the Fahrenheit scale.
ⓓ Graph the equation.
We’ll need to use a larger scale than our usual. Start at the F-intercept (0, 32) , and then count out the rise of 9 and the
run of 5 to get a second point as shown in the graph.

TRY IT : : 3.37

The equation h = 2s + 50 is used to estimate a woman’s height in inches, h, based on her shoe size, s.

ⓐ Estimate the height of a child who wears women’s shoe size 0.


ⓑ Estimate the height of a woman with shoe size 8.
ⓒ Interpret the slope and h-intercept of the equation.
ⓓ Graph the equation.

This OpenStax book is available for free at http://cnx.org/content/col12119/1.5


Chapter 3 Graphs and Functions 267

TRY IT : : 3.38

The equation T = 1 n + 40 is used to estimate the temperature in degrees Fahrenheit, T, based on the number
4
of cricket chirps, n, in one minute.

ⓐ Estimate the temperature when there are no chirps.


ⓑ Estimate the temperature when the number of chirps in one minute is 100.
ⓒ Interpret the slope and T-intercept of the equation.
ⓓ Graph the equation.
The cost of running some types business have two components—a fixed cost and a variable cost. The fixed cost is always
the same regardless of how many units are produced. This is the cost of rent, insurance, equipment, advertising, and
other items that must be paid regularly. The variable cost depends on the number of units produced. It is for the material
and labor needed to produce each item.

EXAMPLE 3.20

Sam drives a delivery van. The equation C = 0.5m + 60 models the relation between his weekly cost, C, in dollars and
the number of miles, m, that he drives.

ⓐ Find Sam’s cost for a week when he drives 0 miles.


ⓑ Find the cost for a week when he drives 250 miles.
ⓒ Interpret the slope and C-intercept of the equation.
ⓓ Graph the equation.
Solution

Find Sam’s cost for a week when he drives 0 miles. C = 0.5m + 60
Find C when m = 0. C = 0.5(0) + 60
Simplify. C = 60
Sam’s costs are $60 when he drives 0 miles.

Find the cost for a week when he drives 250 miles. C = 0.5m + 60
Find C when m = 250. C = 0.5(250) + 60
Simplify. C = 185
Sam’s costs are $185 when he drives 250 miles.
ⓒ Interpret the slope and C-intercept of the equation.

The slope, 0.5, means that the weekly cost, C, increases by $0.50 when the number of miles driven, n, increases by 1.
The C-intercept means that when the number of miles driven is 0, the weekly cost is $60.
ⓓ Graph the equation.
We’ll need to use a larger scale than our usual. Start at the C-intercept (0, 60) .
To count out the slope m = 0.5, we rewrite it as an equivalent fraction that will make our graphing easier.
268 Chapter 3 Graphs and Functions

m = 0.5
Rewrite as a fraction. m = 0.5
1
Multiply numerator and
0.5(100)
denominator by 100. m=
1(100)
Simplify. m= 50
100
So to graph the next point go up 50 from the intercept of 60 and then to the right 100. The second point will be (100, 110).

TRY IT : : 3.39

Stella has a home business selling gourmet pizzas. The equation C = 4p + 25 models the relation between her
weekly cost, C, in dollars and the number of pizzas, p, that she sells.

ⓐ Find Stella’s cost for a week when she sells no pizzas.


ⓑ Find the cost for a week when she sells 15 pizzas.
ⓒ Interpret the slope and C-intercept of the equation.
ⓓ Graph the equation.

TRY IT : : 3.40

Loreen has a calligraphy business. The equation C = 1.8n + 35 models the relation between her weekly cost, C,
in dollars and the number of wedding invitations, n, that she writes.

ⓐ Find Loreen’s cost for a week when she writes no invitations.


ⓑ Find the cost for a week when she writes 75 invitations.
ⓒ Interpret the slope and C-intercept of the equation.
ⓓ Graph the equation.

Use Slopes to Identify Parallel and Perpendicular Lines


Two lines that have the same slope are called parallel lines. Parallel lines have the same steepness and never intersect.
We say this more formally in terms of the rectangular coordinate system. Two lines that have the same slope and different
y-intercepts are called parallel lines. See Figure 3.7.

This OpenStax book is available for free at http://cnx.org/content/col12119/1.5


Chapter 3 Graphs and Functions 269

Figure 3.7

Verify that both lines have the same slope, m = 2 , and different y-intercepts.
5
What about vertical lines? The slope of a vertical line is undefined, so vertical lines don’t fit in the definition above. We say
that vertical lines that have different x-intercepts are parallel, like the lines shown in this graph.

Figure 3.8

Parallel Lines

Parallel lines are lines in the same plane that do not intersect.
• Parallel lines have the same slope and different y-intercepts.
• If m 1 and m 2 are the slopes of two parallel lines then m 1 = m 2.
• Parallel vertical lines have different x-intercepts.

Since parallel lines have the same slope and different y-intercepts, we can now just look at the slope–intercept form of the
equations of lines and decide if the lines are parallel.

EXAMPLE 3.21

Use slopes and y-intercepts to determine if the lines are parallel:


270 Chapter 3 Graphs and Functions

ⓐ 3x − 2y = 6 and y = 3x + 1 ⓑ y = 2x - 3 and −6x + 3y = −9.


2
Solution

3x − 2y = 6 and y = 3x + 1
2
−2y = − 3x + 6
Solve the first equation for y. −2y
= −3x + 6
−2 −2
The equation is now in slope–intercept form. 3
y = x−3
2
The equation of the second line is already
y = 3x + 1
in slope–intercept form. 2

y = 3x − 3 y = 3x + 1
2 2
Identify the slope andy-intercept of both lines. y = mx + b y = mx + b
m =3 y =3
2 2
y-intercept is (0, −3) y-intercept is (0, 1)
The lines have the same slope and different y-intercepts and so they are parallel.
You may want to graph the lines to confirm whether they are parallel.


y = 2x − 3 and −6x + 3y = − 9
The first equation is already in slope–intercept form. y = 2x − 3
−6x + 3y = −9
3y = 6x − 9
Solve the second equation for y. 3y
= 6x − 9
3 3
y = 2x − 3
The second equation is now in slope–intercept form. y = 2x − 3

y = 2x − 3 y = 2x − 3
Identify the slope andy-intercept of both lines. y = mx + b y = mx + b
m =2 m =2
y-intercept is (0, −3) y-intercept is (0, −3)
The lines have the same slope, but they also have the same y-intercepts. Their equations represent the same line and we
say the lines are coincident. They are not parallel; they are the same line.

TRY IT : : 3.41 Use slopes and y-intercepts to determine if the lines are parallel:

ⓐ 2x + 5y = 5 and y = − 2x − 4 ⓑ y = − 12 x − 1 and x + 2y = −2.


5

TRY IT : : 3.42 Use slopes and y-intercepts to determine if the lines are parallel:

ⓐ 4x − 3y = 6 and y = 4x − 1
3
ⓑ y = 34 x − 3 and 3x − 4y = 12.

EXAMPLE 3.22

Use slopes and y-intercepts to determine if the lines are parallel:

This OpenStax book is available for free at http://cnx.org/content/col12119/1.5


Chapter 3 Graphs and Functions 271

ⓐ y = −4 and y=3 ⓑ x = −2 and x = −5.


Solution
ⓐ y = −4 and y=3
We recognize right away from the equations that these are horizontal lines, and so we know their slopes are both 0.
Since the horizontal lines cross the y-axis at y = −4 and at y = 3, we know the y-intercepts are (0, −4) and (0, 3).
The lines have the same slope and different y-intercepts and so they are parallel.
ⓑ x = −2 and x = −5
We recognize right away from the equations that these are vertical lines, and so we know their slopes are undefined.
Since the vertical lines cross the x-axis at x = −2 and x = −5, we know the y-intercepts are (−2, 0) and (−5, 0).
The lines are vertical and have different x-intercepts and so they are parallel.

TRY IT : : 3.43 Use slopes and y-intercepts to determine if the lines are parallel:

ⓐ y=8 and y = −6 ⓑ x=1 and x = −5.

TRY IT : : 3.44 Use slopes and y-intercepts to determine if the lines are parallel:

ⓐ y=1 and y = −5 ⓑ x=8 and x = −6.

Let’s look at the lines whose equations are y = 1 x − 1 and y = −4x + 2, shown in Figure 3.9.
4

Figure 3.9

These lines lie in the same plane and intersect in right angles. We call these lines perpendicular.

If we look at the slope of the first line,m 1 = 1 , and the slope of the second line, m 2 = −4, we can see that they are
4
negative reciprocals of each other. If we multiply them, their product is −1.
m1 · m2
1 (−4)
4
−1
This is always true for perpendicular lines and leads us to this definition.

Perpendicular Lines

Perpendicular lines are lines in the same plane that form a right angle.
272 Chapter 3 Graphs and Functions

• If m 1 and m 2 are the slopes of two perpendicular lines, then:

◦ their slopes are negative reciprocals of each other, m 1 = − m1 .


2

◦ the product of their slopes is −1 , m 1 · m 2 = −1.


• A vertical line and a horizontal line are always perpendicular to each other.

We were able to look at the slope–intercept form of linear equations and determine whether or not the lines were parallel.
We can do the same thing for perpendicular lines.
We find the slope–intercept form of the equation, and then see if the slopes are opposite reciprocals. If the product of the
slopes is −1, the lines are perpendicular.

EXAMPLE 3.23

Use slopes to determine if the lines are perpendicular:

ⓐ y = −5x − 4 and x − 5y = 5 ⓑ 7x + 2y = 3 and 2x + 7y = 5


Solution

The first equation is in slope–intercept form. y = − 5x − 4
x − 5y = 5
−5y = − x + 5
−5y
Solve the second equation for y. = −x + 5
−5 −5
1
y = x−1
5

y = − 5x − 4 y = 1x − 1
5
Identify the slope of each line. y = mx + b y = mx + b
m1 = − 5 m2 = 1
5
The slopes are negative reciprocals of each other, so the lines are perpendicular. We check by multiplying the slopes,
⎛ ⎞
Since −5⎝1 ⎠ = −1, it checks.
5


7x + 2y = 3 2x + 7y = 5
2y = − 7x + 3 7y = − 2x + 5
2y 7y
Solve the equations for y. = −7x + 3 = −2x + 5
2 2 7 7
y = − x+3
7 y = − x+5
2
2 2 7 7
y = mx + b y = mx + b
Identify the slope of each line.
m1 = − 7 m1 = − 2
2 7
The slopes are reciprocals of each other, but they have the same sign. Since they are not negative reciprocals, the lines
are not perpendicular.

TRY IT : : 3.45 Use slopes to determine if the lines are perpendicular:

ⓐ y = −3x + 2 and x − 3y = 4 ⓑ 5x + 4y = 1 and 4x + 5y = 3.

This OpenStax book is available for free at http://cnx.org/content/col12119/1.5


Chapter 3 Graphs and Functions 273

TRY IT : : 3.46 Use slopes to determine if the lines are perpendicular:

ⓐ y = 2x − 5 and x + 2y = −6 ⓑ 2x − 9y = 3 and 9x − 2y = 1.
274 Chapter 3 Graphs and Functions

3.2 EXERCISES
Practice Makes Perfect
Find the Slope of a Line
In the following exercises, find the slope of each line shown.
73. 74.

75. 76.

This OpenStax book is available for free at http://cnx.org/content/col12119/1.5


Chapter 3 Graphs and Functions 275

77. 78.

79. 80.

In the following exercises, find the slope of each line.


81. y=3 82. y = −2 83. x = −5

84. x=4

In the following exercises, use the slope formula to find the slope of the line between each pair of points.
85. (2, 5), (4, 0) 86. (3, 6), (8, 0) 87. (−3, 3), (4, −5)

88. (−2, 4), (3, −1) 89. (−1, −2), (2, 5) 90. (−2, −1), (6, 5)

91. (4, −5), (1, −2) 92. (3, −6), (2, −2)
276 Chapter 3 Graphs and Functions

Graph a Line Given a Point and the Slope


In the following exercises, graph each line with the given point and slope.

93. (2, 5); m = − 1 94. (1, 4) ; m = − 1 95. (−1, −4) ; m = 4


3 2 3

96. (−3, −5) ; m = 3 97. y-intercept 3; m= −2 98. x-intercept −2 ; m = 3


2 5 4

99. (−4, 2) ; m = 4 100. (1, 5) ; m = −3

Graph a Line Using Its Slope and Intercept


In the following exercises, identify the slope and y-intercept of each line.
101. y = −7x + 3 102. y = 4x − 10 103. 3x + y = 5

104. 4x + y = 8 105. 6x + 4y = 12 106. 8x + 3y = 12

107. 5x − 2y = 6 108. 7x − 3y = 9

In the following exercises, graph the line of each equation using its slope and y-intercept.
109. y = 3x − 1 110. y = 2x − 3 111. y = −x + 3

112. y = −x − 4 113. y = − 2x − 3 114. y = − 3x + 2


5 5

115. 3x − 2y = 4 116. 3x − 4y = 8

Choose the Most Convenient Method to Graph a Line


In the following exercises, determine the most convenient method to graph each line.
117. x=2 118. y=5 119. y = −3x + 4

120. x−y=5 121. x−y=1 122. y = 2x − 1


3

123. 3x − 2y = −12 124. 2x − 5y = −10

This OpenStax book is available for free at http://cnx.org/content/col12119/1.5


Chapter 3 Graphs and Functions 277

Graph and Interpret Applications of Slope–Intercept


125. The equation 126. The equation 127. Bruce drives his car for his
P = 31 + 1.75w models the P = 28 + 2.54w models the job. The equation
relation between the amount of relation between the amount of R R = 0.575m + 42 models the
Tuyet’s monthly water bill and y’s monthly water bill relation between the amount in
payment, P, in dollars, and the payment, P, in dollars, and the dollars, R, that he is reimbursed
number of units of water, w, used. number of units of water, w, used. and the number of miles, m, he
drives in one day.
ⓐ Find Tuyet’s payment for a ⓐ Find the payment for a month
month when 0 units of water are when R and y used 0 units of ⓐ Find the amount Bruce is
used. water. reimbursed on a day when he
drives 0 miles.
ⓑ Find Tuyet’s payment for a ⓑ Find the payment for a month
month when 12 units of water are when R and y used 15 units of ⓑ Find the amount Bruce is
used. water. reimbursed on a day when he
drives 220 miles.
ⓒ Interpret the slope and ⓒ Interpret the slope and
P-intercept of the equation. P-intercept of the equation. ⓒ Interpret the slope and
R-intercept of the equation.
ⓓ Graph the equation. ⓓ Graph the equation.
ⓓ Graph the equation.

128. Janelle is planning to rent a 129. Cherie works in retail and her 130. Patel’s weekly salary includes
car while on vacation. The weekly salary includes a base pay plus commission on his
equation C = 0.32m + 15 commission for the amount she sales. The equation
models the relation between the sells. The equation S = 750 + 0.09c models the
cost in dollars, C, per day and the S = 400 + 0.15c models the relation between his weekly
number of miles, m, she drives in relation between her weekly salary, S, in dollars and the
one day. salary, S, in dollars and the amount of his sales, c, in dollars.
amount of her sales, c, in dollars.
ⓐ Find the cost if Janelle drives ⓐ Find Patel’s salary for a week
the car 0 miles one day. ⓐ Find Cherie’s salary for a week when his sales were 0.
when her sales were $0.
ⓑ Find the cost on a day when ⓑ Find Patel’s salary for a week
Janelle drives the car 400 miles. ⓑ Find Cherie’s salary for a week when his sales were 18,540.
when her sales were $3,600.
ⓒ Interpret the slope and ⓒ Interpret the slope and
C-intercept of the equation. ⓒ Interpret the slope and S-intercept of the equation.
S-intercept of the equation.
ⓓ Graph the equation. ⓓ Graph the equation.
ⓓ Graph the equation.

131. Costa is planning a lunch 132. Margie is planning a dinner


banquet. The equation banquet. The equation
C = 450 + 28g models the C = 750 + 42g models the
relation between the cost in relation between the cost in
dollars, C, of the banquet and the dollars, C of the banquet and the
number of guests, g. number of guests, g.

ⓐ Find the cost if the number of ⓐ Find the cost if the number of
guests is 40. guests is 50.

ⓑ Find the cost if the number of ⓑ Find the cost if the number of
guests is 80. guests is 100.

ⓒ Interpret the slope and ⓒ Interpret the slope and


C-intercept of the equation. C-intercept of the equation.

ⓓ Graph the equation. ⓓ Graph the equation.


278 Chapter 3 Graphs and Functions

Use Slopes to Identify Parallel and Perpendicular Lines


In the following exercises, use slopes and y-intercepts to determine if the lines are parallel, perpendicular, or neither.
133. 134. 135. 2x − 4y = 6; x − 2y = 3
y = 3 x − 3; 3x − 4y = −2 3x − 4y = − 2; y = 3x − 3
4 4

136. 137. x = 5; x = −6 138. x = − 3; x= −2


8x + 6y = 6; 12x + 9y = 12

139. 4x − 2y = 5; 3x + 6y = 8 140. 141.


8x − 2y = 7; 3x + 12y = 9 3x − 6y = 12; 6x − 3y = 3

142. 143. 144. 5x − 2y = 11; 5x − y = 7


9x − 5y = 4; 5x + 9y = − 1 7x − 4y = 8; 4x + 7y = 14

145. 3x − 2y = 8; 2x + 3y = 6 146. 2x + 3y = 5; 3x − 2y = 7 147. 3x − 2y = 1; 2x − 3y = 2

148. 2x + 4y = 3; 6x + 3y = 2 149. y = 2; y=6 150. y = − 1; y=2

Writing Exercises

151. How does the graph of a line with slope m=1 152. Why is the slope of a vertical line “undefined”?
2
differ from the graph of a line with slope m = 2 ?

153. Explain how you can graph a line given a point and 154. Explain in your own words how to decide which
its slope. method to use to graph a line.

Self Check

ⓐ After completing the exercises, use this checklist to evaluate your mastery of the objectives of this section.

ⓑ After reviewing this checklist, what will you do to become confident for all objectives?

This OpenStax book is available for free at http://cnx.org/content/col12119/1.5


Chapter 3 Graphs and Functions 279

3.3
Find the Equation of a Line
Learning Objectives
By the end of this section, you will be able to:
Find an equation of the line given the slope and y-intercept
Find an equation of the line given the slope and a point
Find an equation of the line given two points
Find an equation of a line parallel to a given line
Find an equation of a line perpendicular to a given line

Be Prepared!

Before you get started, take this readiness quiz.

1. Solve: 2 (x + 15).
5
If you missed this problem, review Example 1.50.
2. Simplify: −3⎛⎝x − (−2)⎞⎠.
If you missed this problem, review Example 1.53.
3. Solve for y: y − 3 = −2(x + 1).
If you missed this problem, review Example 2.31.

How do online companies know that “you may also like” a particular item based on something you just ordered? How
can economists know how a rise in the minimum wage will affect the unemployment rate? How do medical researchers
create drugs to target cancer cells? How can traffic engineers predict the effect on your commuting time of an increase
or decrease in gas prices? It’s all mathematics.
The physical sciences, social sciences, and the business world are full of situations that can be modeled with linear
equations relating two variables. To create a mathematical model of a linear relation between two variables, we must be
able to find the equation of the line. In this section, we will look at several ways to write the equation of a line. The specific
method we use will be determined by what information we are given.
Find an Equation of the Line Given the Slope and y-Intercept
We can easily determine the slope and intercept of a line if the equation is written in slope-intercept form, y = mx + b.
Now we will do the reverse—we will start with the slope and y-intercept and use them to find the equation of the line.

EXAMPLE 3.24

Find the equation of a line with slope −9 and y-intercept (0, −4).
Solution
Since we are given the slope and y-intercept of the line, we can substitute the needed values into the slope-intercept form,
y = mx + b.

Name the slope.

Name the y-intercept.

Substitute the values into y = mx + b.

TRY IT : : 3.47
Find the equation of a line with slope 2 and y-intercept (0, 4).
5
280 Chapter 3 Graphs and Functions

TRY IT : : 3.48 Find the equation of a line with slope −1 and y-intercept (0, −3).

Sometimes, the slope and intercept need to be determined from the graph.

EXAMPLE 3.25

Find the equation of the line shown in the graph.

Solution
We need to find the slope and y-intercept of the line from the graph so we can substitute the needed values into the
slope-intercept form, y = mx + b.

To find the slope, we choose two points on the graph.


The y-intercept is (0, −4) and the graph passes through (3, −2).

Find the slope, by counting the rise and run.

Find the y-intercept.

Substitute the values into y = mx + b.

This OpenStax book is available for free at http://cnx.org/content/col12119/1.5


Chapter 3 Graphs and Functions 281

TRY IT : : 3.49 Find the equation of the line shown in the graph.

TRY IT : : 3.50 Find the equation of the line shown in the graph.

Find an Equation of the Line Given the Slope and a Point


Finding an equation of a line using the slope-intercept form of the equation works well when you are given the slope and
y-intercept or when you read them off a graph. But what happens when you have another point instead of the y-intercept?
We are going to use the slope formula to derive another form of an equation of the line.
Suppose we have a line that has slope m and that contains some specific point (x 1, y 1) and some other point, which we
will just call (x, y). We can write the slope of this line and then change it to a different form.
y−y
m = x − x1
1
⎛y − y 1 ⎞
Multiply both sides of the equation by x − x 1. m(x − x 1) = ⎝ x − x ⎠(x − x 1)
1
Simplify. m(x − x 1) = y − y 1
Rewrite the equation with the y terms on the left. y − y 1 = m(x − x 1)
This format is called the point-slope form of an equation of a line.
282 Chapter 3 Graphs and Functions

Point-slope Form of an Equation of a Line

The point-slope form of an equation of a line with slope m and containing the point (x 1, y 1) is:
y − y 1 = m(x − x 1)
We can use the point-slope form of an equation to find an equation of a line when we know the slope and at least one
point. Then, we will rewrite the equation in slope-intercept form. Most applications of linear equations use the the slope-
intercept form.

EXAMPLE 3.26 HOW TO FIND AN EQUATION OF A LINE GIVEN A POINT AND THE SLOPE

Find an equation of a line with slope m = − 1 that contains the point (6, −4). Write the equation in slope-intercept
3
form.
Solution

TRY IT : : 3.51
Find the equation of a line with slope m = − 2 and containing the point (10, −5).
5

TRY IT : : 3.52
Find the equation of a line with slope m = − 3 , and containing the point (4, −7).
4

We list the steps for easy reference.

HOW TO : : TO FIND AN EQUATION OF A LINE GIVEN THE SLOPE AND A POINT.

Step 1. Identify the slope.


Step 2. Identify the point.
Step 3. Substitute the values into the point-slope form, y − y 1 = m(x − x 1).
Step 4. Write the equation in slope-intercept form.

EXAMPLE 3.27

Find an equation of a horizontal line that contains the point (−2, −6). Write the equation in slope-intercept form.
Solution
Every horizontal line has slope 0. We can substitute the slope and points into the point-slope form, y − y 1 = m(x − x 1).

This OpenStax book is available for free at http://cnx.org/content/col12119/1.5


Chapter 3 Graphs and Functions 283

Identify the slope.

Identify the point.

Substitute the values into y − y 1 = m(x − x 1).

Simplify.

Write in slope-intercept form. It is in y-form, but could be written y = 0x − 6.

Did we end up with the form of a horizontal line, y = a?

TRY IT : : 3.53 Find the equation of a horizontal line containing the point (−3, 8).

TRY IT : : 3.54 Find the equation of a horizontal line containing the point (−1, 4).

Find an Equation of the Line Given Two Points


When real-world data is collected, a linear model can be created from two data points. In the next example we’ll see how
to find an equation of a line when just two points are given.
So far, we have two options for finding an equation of a line: slope-intercept or point-slope. When we start with two points,
it makes more sense to use the point-slope form.
But then we need the slope. Can we find the slope with just two points? Yes. Then, once we have the slope, we can use it
and one of the given points to find the equation.

EXAMPLE 3.28 HOW TO FIND THE EQUATION OF A LINE GIVEN TWO POINTS

Find an equation of a line that contains the points (−3, −1) and (2, −2) Write the equation in slope-intercept form.
Solution
284 Chapter 3 Graphs and Functions

TRY IT : : 3.55 Find the equation of a line containing the points (−2, −4) and (1, −3).

TRY IT : : 3.56 Find the equation of a line containing the points (−4, −3) and (1, −5).

The steps are summarized here.

HOW TO : : TO FIND AN EQUATION OF A LINE GIVEN TWO POINTS.

Step 1. y −y
Find the slope using the given points. m = x2 − x1
2 1

Step 2. Choose one point.


Step 3. Substitute the values into the point-slope form: y − y 1 = m(x − x 1).
Step 4. Write the equation in slope-intercept form.

EXAMPLE 3.29

Find an equation of a line that contains the points (−3, 5) and (−3, 4). Write the equation in slope-intercept form.
Solution
Again, the first step will be to find the slope.
y −y
Find the slope of the line through (−3, 5) and (−3, 4). m = x2 − x1
2 1
m = 4 − 5
−3 − (−3)
m = −1
0
The slope is undefined.
This tells us it is a vertical line. Both of our points have an x-coordinate of −2. So our equation of the line is x = −2.
Since there is no y, we cannot write it in slope-intercept form.
You may want to sketch a graph using the two given points. Does your graph agree with our conclusion that this is a
vertical line?

TRY IT : : 3.57 Find the equation of a line containing the points (5, 1) and (5, −4).

TRY IT : : 3.58 Find the equaion of a line containing the points (−4, 4) and (−4, 3).

We have seen that we can use either the slope-intercept form or the point-slope form to find an equation of a line. Which
form we use will depend on the information we are given.

This OpenStax book is available for free at http://cnx.org/content/col12119/1.5


Chapter 3 Graphs and Functions 285

To Write an Equation of a Line

If given: Use: Form:

Slope and y-intercept slope-intercept y = mx + b

Slope and a point point-slope y − y 1 = m(x − x 1)

Two points point-slope y − y 1 = m(x − x 1)

Find an Equation of a Line Parallel to a Given Line


Suppose we need to find an equation of a line that passes through a specific point and is parallel to a given line. We can
use the fact that parallel lines have the same slope. So we will have a point and the slope—just what we need to use the
point-slope equation.
First, let’s look at this graphically.
This graph shows y = 2x − 3. We want to graph a line parallel to this line and passing through the point (−2, 1).

We know that parallel lines have the same slope. So the second line will have the same slope as y = 2x − 3. That slope
is m∥ = 2. We’ll use the notation m∥ to represent the slope of a line parallel to a line with slope m. (Notice that the
subscript || looks like two parallel lines.)
The second line will pass through (−2, 1) and have m = 2.
To graph the line, we start at (−2, 1) and count out the rise and run.

With m = 2 (or m = 2 ), we count out the rise 2 and the run 1. We draw the line, as shown in the graph.
1

Do the lines appear parallel? Does the second line pass through (−2, 1) ?
We were asked to graph the line, now let’s see how to do this algebraically.
286 Chapter 3 Graphs and Functions

We can use either the slope-intercept form or the point-slope form to find an equation of a line. Here we know one point
and can find the slope. So we will use the point-slope form.

EXAMPLE 3.30 HOW TO FIND THE EQUATION OF A LINE PARALLEL TO A GIVEN LINE AND A POINT

Find an equation of a line parallel to y = 2x − 3 that contains the point (−2, 1). Write the equation in slope-intercept
form.
Solution

Look at graph with the parallel lines shown previously. Does this equation make sense? What is the y-intercept of the line?
What is the slope?

TRY IT : : 3.59

Find an equation of a line parallel to the line y = 3x + 1 that contains the point (4, 2). Write the equation in
slope-intercept form.

TRY IT : : 3.60
Find an equation of a line parallel to the line y = 1 x − 3 that contains the point (6, 4).
2
Write the equation in slope-intercept form.

HOW TO : : FIND AN EQUATION OF A LINE PARALLEL TO A GIVEN LINE.

Step 1. Find the slope of the given line.


Step 2. Find the slope of the parallel line.
Step 3. Identify the point.
Step 4. Substitute the values into the point-slope form: y − y 1 = m(x − x 1).
Step 5. Write the equation in slope-intercept form.

Find an Equation of a Line Perpendicular to a Given Line


Now, let’s consider perpendicular lines. Suppose we need to find a line passing through a specific point and which is
perpendicular to a given line. We can use the fact that perpendicular lines have slopes that are negative reciprocals. We
will again use the point-slope equation, like we did with parallel lines.
This graph shows y = 2x − 3. Now, we want to graph a line perpendicular to this line and passing through (−2, 1).

This OpenStax book is available for free at http://cnx.org/content/col12119/1.5


Chapter 3 Graphs and Functions 287

We know that perpendicular lines have slopes that are negative reciprocals.
We’ll use the notation m⊥ to represent the slope of a line perpendicular to a line with slope m. (Notice that the subscript
⊥ looks like the right angles made by two perpendicular lines.)
y = 2x − 3 perpendicular line
m=2 m⊥ = − 1
2

We now know the perpendicular line will pass through (−2, 1) with m⊥ = − 1 .
2
To graph the line, we will start at (−2, 1) and count out the rise −1 and the run 2. Then we draw the line.

Do the lines appear perpendicular? Does the second line pass through (−2, 1) ?
We were asked to graph the line, now, let’s see how to do this algebraically.
We can use either the slope-intercept form or the point-slope form to find an equation of a line. In this example we know
one point, and can find the slope, so we will use the point-slope form.

EXAMPLE 3.31 HOW TO FIND THE EQUATION OF A LINE PERPENDICULAR TO A GIVEN LINE AND A POINT

Find an equation of a line perpendicular to y = 2x − 3 that contains the point (−2, 1). Write the equation in slope-
intercept form.
Solution
288 Chapter 3 Graphs and Functions

TRY IT : : 3.61

Find an equation of a line perpendicular to the line y = 3x + 1 that contains the point (4, 2). Write the equation
in slope-intercept form.

TRY IT : : 3.62

Find an equation of a line perpendicular to the line y = 1 x − 3 that contains the point (6, 4). Write the equation
2
in slope-intercept form.

HOW TO : : FIND AN EQUATION OF A LINE PERPENDICULAR TO A GIVEN LINE.

Step 1. Find the slope of the given line.


Step 2. Find the slope of the perpendicular line.
Step 3. Identify the point.
Step 4. Substitute the values into the point-slope form, y − y 1 = m(x − x 1).
Step 5. Write the equation in slope-intercept form.

EXAMPLE 3.32

Find an equation of a line perpendicular to x = 5 that contains the point (3, −2). Write the equation in slope-intercept
form.
Solution
Again, since we know one point, the point-slope option seems more promising than the slope-intercept option. We
need the slope to use this form, and we know the new line will be perpendicular to x = 5. This line is vertical, so its
perpendicular will be horizontal. This tells us the m⊥ = 0.

This OpenStax book is available for free at http://cnx.org/content/col12119/1.5


Chapter 3 Graphs and Functions 289

Identify the point. (3, −2)


Identify the slope of the perpendicular line. m⊥ = 0
Substitute the values into y − y 1 = m(x − x 1). y − y 1 = m(x − x 1)
y − (−2) = 0(x − 3)
Simplify. y+2 = 0
y = −2
Sketch the graph of both lines. On your graph, do the lines appear to be perpendicular?

TRY IT : : 3.63

Find an equation of a line that is perpendicular to the line x = 4 that contains the point (4, −5). . Write the
equation in slope-intercept form.

TRY IT : : 3.64

Find an equation of a line that is perpendicular to the line x = 2 that contains the point (2, −1). Write the
equation in slope-intercept form.

In Example 3.32, we used the point-slope form to find the equation. We could have looked at this in a different way.
We want to find a line that is perpendicular to x = 5 that contains the point (3, −2). This graph shows us the line x = 5
and the point (3, −2).

We know every line perpendicular to a vertical line is horizontal, so we will sketch the horizontal line through (3, −2).
290 Chapter 3 Graphs and Functions

Do the lines appear perpendicular?


If we look at a few points on this horizontal line, we notice they all have y-coordinates of −2. So, the equation of the line
perpendicular to the vertical line x = 5 is y = −2.

EXAMPLE 3.33

Find an equation of a line that is perpendicular to y = −3 that contains the point (−3, 5). Write the equation in slope-
intercept form.
Solution
The line y = −3 is a horizontal line. Any line perpendicular to it must be vertical, in the form x = a. Since the
perpendicular line is vertical and passes through (−3, 5), every point on it has an x-coordinate of −3. The equation of
the perpendicular line is x = −3
You may want to sketch the lines. Do they appear perpendicular?

TRY IT : : 3.65

Find an equation of a line that is perpendicular to the line y = 1 that contains the point (−5, 1). Write the
equation in slope-intercept form.

TRY IT : : 3.66

Find an equation of a line that is perpendicular to the line y = −5 that contains the point (−4, −5). Write the
equation in slope-intercept form.

MEDIA : :
Access these online resources for additional instruction and practice with finding the equation of a line.
• Write an Equation of Line Given its slope and Y-Intercept (https://openstax.org/l/37slopeycept)
• Using Point Slope Form to Write the Equation of a Line, Find the equation given slope and point
(https://openstax.org/l/37slopepoint)
• Find the equation given two points (https://openstax.org/l/37twoptspline)
• Find the equation of perpendicular and parallel lines (https://openstax.org/l/37perpenpara)

This OpenStax book is available for free at http://cnx.org/content/col12119/1.5


Chapter 3 Graphs and Functions 291

3.3 EXERCISES
Practice Makes Perfect
Find an Equation of the Line Given the Slope and y-Intercept
In the following exercises, find the equation of a line with given slope and y-intercept. Write the equation in slope-intercept form.
155. slope 3 and 156. slope 8 and 157. slope −3 and
y -intercept (0, 5) y-intercept (0, −6) y -intercept (0, −1)

158. slope −1 and 159. slope 1 and 160. slope − 3 and


y -intercept (0, 3) 5 4
y -intercept (0, −5) y -intercept (0, −2)

161. slope 0 and 162. slope −4 and


y -intercept (0, −1) y -intercept (0, 0)

In the following exercises, find the equation of the line shown in each graph. Write the equation in slope-intercept form.
163. 164.

165. 166.
292 Chapter 3 Graphs and Functions

167. 168.

169. 170.

Find an Equation of the Line Given the Slope and a Point


In the following exercises, find the equation of a line with given slope and containing the given point. Write the equation in slope-
intercept form.

171. m = 5 , point (8, 3) 172. m = 5 , point (6, 7) 173. m = − 3 , point (10, −5)
8 6 5

174. m = − 3 , point (8, −5) 175. m = − 3 , point (−4, −3) 176. m = − 5 , point (−8, −2)
4 2 2

177. m = −7, point (−1, −3) 178. m = −4, point (−2, −3) 179. Horizontal line containing
(−2, 5)

180. Horizontal line containing 181. Horizontal line containing 182. Horizontal line containing
(−2, −3) (−1, −7) (4, −8)

This OpenStax book is available for free at http://cnx.org/content/col12119/1.5

You might also like